Sunteți pe pagina 1din 54

LECTURE NOTES IN FLUID MACHINERY (ME 413 )

Pump-is a machine used to add energy to a liquid in order to transfer the liquid from one point to another point
of higher energy level. A pump is a machine that converts mechanical energy into fluid energy.

Function of a pump is to add to the pressure existing on a liquid an increment sufficient to the required service.
This service maybe the production of a velocity or the overcoming of friction or external pressure.

Purpose: is to move a quantity of water against a pressure

At ordinary temperatures; 1000 kg = 1 m3, or 1 kg= 1 litre, 1 kg/cm2 = 10 m. Note; This could not be used in hot
water.

Uses of water pumps

1. Condensing water
2. Condensate
3. Boiler feed
4. Heater drains
5. Booster flows
6. Sump drain and other services

FIGURE: TYPICAL PUMP INSTALLATION

Upper
Reservoir
Discharge Pipe

Pressure Gauge
Gate Valve
Pump Check Valve
Suction Pipe

Foot Valve w/ Strainer


Lower Reservoir

CLASSIFICATION OF PUMPS

1. Centrifugal pump- is a machine which the pumping action is accomplished by imparting kinetic energy
to the fluid by a high speed revolving impeller with vanes and subsequently converting this kinetic energy
into pressure energy either by passing through a volute casing or through diffuser vanes. It is high
discharge, low head, high speed, not self priming.
2. Rotary Pump- a positive displacement pump consisting of a fixed casing containing gears, cams, screws,
vanes plungers or similar elements actuated by rotation of the drive shaft. It is low discharge, low head,
used for pumping viscous liquids like oil. Example; Gear pump, Screw pump, vane Pump
3. Reciprocating pump- is a positive displacement unit wherein the pumping action is accomplished by the
forward and backward movement of a piston or plunger provided with valves. It is low discharge, high
head, low speed, self- priming.4.0) Deep well pump- is divided into plunger or reciprocating, turbine,
ejector-centrifugal types and air lifts. Turbine pumps-are used for pumping water with high suction lift,
for pumping condensate. Jet Pump or Injector pump- used for pumping boiler feed water, used as
accessory of centrifugal pump.
Types of pumps are classified into two major category.
1. Dynamic or kinetic are types of pumps in which energy is continuously added to the fluid to
increase its velocity. Examples; centrifugal, jet and turbine pumps
2. Positive displacement pumps-are types of pumps, in which energy is continuously added by
application of force to an enclosed volume of fluid and resulting to a direct increase in its pressure.
Examples: Reciprocating, rotary, and diaphragm pumps.
Some other ways of classifying pumps includes the following;
1. Type of flow of fluid at the impeller; radial semi-axial, or axial
2. Type of casing; volute or diffuser pumps
3. Type of design: single stage, multi-stage, single suction, dual suction, horizontal, vertical pump,
submersible.
4. Type of application: fire, dredge, slurry, deep well, irrigation, drainage, circulating, boiler feed,
condensate, clogless.
CENTRIFUGAL PUMP
A centrifugal pump consist of stationary casing and an impeller connected in rotating shaft. Liquid enters
the centre of the rotating impeller and leaves at a high velocity and passes to a stationary volute casing
which transforms kinetic energy into pressure. The term centrifugal came from the centrifugal force
created as the water move outward from the centre of impeller rotation. A centrifugal pump whose
drive unit is supported by the pump having its suction and discharge flanges on approximately the same
order is called in line pump. A centrifugal pump characterized by a housing which is split parallel to the
shaft is called horizontal split case pump. A centrifugal pump with one or more impellers discharging
into one or more bowls and a vertical ejector or column pipe used to connect the bowls to the discharge
heads on which the pump driver is mounted is called vertical shaft turbine pump.
FIGURE: Impeller

Casing

Vane or
Blade
Advantages of centrifugal pumps
1. Simple and compact
2. Easy to maintain
3. Adaptability with motor with high rpm
4. Little vibrations
5. Flow can be controlled from full to non-discharge without shutting the pump

Disadvantages

1. Poor suction power


2. Usually needs priming
3. Cavitation may develop during operation
4. Needs multi-stage to increase discharge pressure
5. Cannot handle very viscous fluid
6. Check valve is required to avoid back flow.

One of the disadvantages of a centrifugal pump is that the liquid is delivered at a relatively low pressure.
This due to the high velocity acquired by the liquid as it leaves the impeller. Therefore, liquids are
delivered at low pressure but at a high velocity. In order, to increase the discharge pressure of liquids, a
decrease in its velocity at the impeller discharge is required. This is usually done by providing additional
impellers to the pump. The liquid exits the first impeller at high velocity then passes thru a series of
impellers thus, additional pressure is added.

Parts of centrifugal pump

1. The shaft- is a piece of metal where the internal parts of the pump are mounted. Its function is to
transmit torque from the motor to the rotating parts of the pump.
Pump shaft has also a shaft sleeve, it is a metal cylinder that fits over the shaft to protect it from
corrosion, erosion, and wear.
2. The impellers of a centrifugal pumps are the most critical part of the pump, because the capacity of
the pump depends on its diameter and speed of which it turns. Impellers can be open, semi- open
and closed type impellers.
FIGURE:
3. The pump casing- is the stationary part of a centrifugal pump and it can either be volute or circular.
The main function of the volute is to convert the kinetic energy acquired by the fluid into pressure.
It could be single and double volute casing. Diffuser volute design has a set of fixed vanes in its casing
that gradually reduces fluid velocity. It gives direction to the flow from the impeller and converts
this velocity energy into pressure energy
4. Wear rings-are fitted to the impellers or to the pump casing to protect the actual impeller and casing
from wear. Excessive wear in the pump will reduce its efficiency particularly in small pumps used in
high speed applications. Wear rings should be replaced periodically to avoid damage in the impeller
or in the casing. It keeps internal recirculation down to a minimum.
5. Packing or mechanical seal- the main function of packing or mechanical seal is to protect the pump
from leakage in the area where the shaft passes through the casing. The most common means of
throttling the leakage between the inside and outside of the casing.
6. Bearings-are used to keep the correct alignment of the shaft with other stationary parts of the pump.
It accurately locate shaft and carry radial and thrust loads.
7. Labyrinth and seals and deflectors- the main purpose is to retain oil in the pump housing and to
prevent foreign materials entering the pump housing.

HEAD and POWER CALCULATIONS

FIGURE:
Motor
Static Discharge Total Static
Head Head

Pump

Pump
Center Line

Static
Sunction

Discharge-volume flow rate of liquid handled by the pump in m3/s or gal/min


Head- total energy developed by the pump, expressed in height of the liquid in meters.
Basic Principles:
General Flow Equation: Q= A v or v=Q/A
H= total head or total dynamic head (TDH)
2 2
H= ( zd zs ) +
+ (hfs + hfd ) + 2
Note:
Zs is negative if source is below the pump centre line
Ps is negative if it is vacuum
Static head- is the height of the surface of the water above the gauge point.
Velocity head-is the head required to produce the flow of water.
Pressure head= is the static head plus gauge pressure on the water surface plus friction head
Dynamic head= is the pressure head plus velocity head.
Pump operating head-is the algebraic difference of discharge and suction head. This can also be
called as total dynamic head.
WATER POWER OR HYDRAULIC POWER- is the theoretical power necessary to raise a given volume
of liquid from a lower to a higher elevation.

Water Power= Q w H; kW
Where;
Q = discharge, m3/s
W= specific weight=9.81 kN/m3 for water
H= total head, in meters

Brake (Input) Power =
Motor efficiency is the ratio of brake power to input power.

em =
Overall efficiency is the ratio of hydraulic power or water power to the input power.

e=
Head, as determined from readings of pressure gauges

2 2
H= +
2
Note: Ps is negative if a vacuum

Calculating the friction head


Friction head is the head lost by the flow in a stream or conduit due to frictional disturbances set
up by the moving liquid and its containing conduit and by intermolecular actions.
2
hf = 2
, (Darcy equation)
2 2
hf=
, ( Morse equation)
where;
hf = friction head loss, m
f = coefficient of friction ( should be taken from Morse table if Morse equation is used)
L= total length, m (including equivalent lengths of the fittings)
V= velocity, m/s
g = 9.81 m/s2
D= inside diameter, m

Example Problems:
1. Water in the rural areas is often extracted from underground water source whose free surface
is 60 m below ground level. The water is to be raised 5m above the ground by a pump. The
diameter of the pipe is 10 cm at the inlet and 15 cm at the exit. Neglecting any heat interaction
with the surroundings and frictional heating effect, what is the necessary power input for the
steady flow of water at the rate of 15 litres/ sec in kW?
Solution:
FIGURE:

5m 15cm

60m 10cm

Q=15 litres/s = 0.015 m3/s Water Source


4(0.015)
Vs = = (0.10)2 = 1.91 m/s
0.015
Vd = (0.15)2
=0.85 m/s
4
2 2 0.85)2 (1.91)2
H=( Zd Zs ) + 2
= 5-(-60 ) +( 2(9.81)
=64.85 m
WP= Q w H =0.015 (9.81) (64.85) = 9.54 kW

2. Calculate the drive horsepower for pumping 1703 L/min cold water to a tank suction at 127 mm
Hg vacuum, delivery at 5.3 kg/cm2 ga., both measured close to pump, efficiency of the pump is
0.65.

Solution: 5.3
2
Figure:
Pump

127mmHgvac
Let;
hd= total head at discharge
hs =total head at suction
h= pump head
By Bernoullis Equation
2 2
H= hd hs =
+ 2
+ (Zd Zs ) +( hfd + hfs)
Where; Pd= 5.3 kg/cm2ga
Ps=-127 mmHg = -0.1727 kg/cm2
(5.3+0.1721) .(100 .
H= 1000 .
= 54.72 m
Pbrake =wQh
=1000kg/m3(1703 L/min) (1m3/1000L) (54.72)
=93,188.16 kg-m/min ( I kW/6116.3 kg-m/min)=15.24 kW=20.42 hp
Drive hp of the pump;
20.42
Hp= = 31.42 hp
0.65
3. Water from a well is to be pumped to a height of 40 meters from the source. The quantity of
water to be delivered is 500 L/s, through a 480 mm diameter pipe. It is also desired to maintain
a 100 kPa pressure at the summit. The frictional head losses from the suction and discharge of
the pump are equivalent to 2.0 m. If the pump speed is 850 rpm, determine the power needed
by the pump in hp.
Solution: 100kPag
Figure:

40m

Water Source

Solving for the water Power


Source: suction lift
WP = hs + hv + hP + hL
hs =40 m
Solving for the velocity at discharge;
Q= A2 v2
(0.480)2
0.5 m3/s = 4
(v2)
v2 = 2.7631 m/s
Neglecting the velocity at suction;
22 12 (2.7631)2 02
hv = 2
= 2(9.81) =0.3891 m
(1000)
hf = 2 1 = 9.81 = 10.1937 m
hl = 2.0 m
TDH = 40m +0.3891 +10.1937 + 2.0 m = 52.5828 m
Thus,
WP= 0.5 m3/s (9.81 kN/m3) (52.5828m) = 257.9186 kW
WP = 257.9186 kW x 1 hp/0.746 kW =345.7354 hp

4. A water source with an elevation of 10 m is to be pumped by a centrifugal pump to an open


reservoir at an elevation of 80 m. The suction pipe and discharge pipe diameter is 255mm and
200 mm, respectively. The pump centreline is at 5m. The head loss at suction pipe is 0.05m and
0.45 at the discharge pipe. The pump delivers water at 15 L/s and has an efficiency of 75% while
the electric motor is 80%. Determine:
a. Water horsepower of the pump
b. Brake horsepower of the pump
c. Power input to drive the motor, in kW
d. The pressure gage readings installed before and after the suction and discharge of the
pump, kPa.
Solution;
Figure: B

Reserve
Oil

70m 200mm

Source

80m

10m
2

1 225mm

5m

Solving for WP
Source: Suction head
WP =QTDH; TDH = hs + hv + hf + Hl
Solving for the water velocity at the suction and discharge
Q= Av= A1v1 = A2v2
(0.255)2
0.015 = V1 ; V1 = 0.2937 m/s
4
(0.200)2
0.015 m3/s = 4
v2 ; v2 = 0.4775 m/s
2 2
(0.4775) (0.2937)2
2
hv = 22 1 = 2(9.81)
=0.0072246 m
Since both the source and the reservoir are open to the atmosphere
Hp =0
HL =0.05m + 0.45 m =0.5 m
TDH=70m + 7.2246 x 10-3 +0 +0.5 =70.5072
Thus,
WP = 0.015 m3/s (9.81)kN/m3 (70.5072 m) = 10.3751 kW ; Thus;
1
Whp =10.3751 kW x =13.9077 hp
0.746
Solving for the brake horsepower of the pump;

Np =
13.9077
0.75 =

Bhp = 18 5436 hp

Solving for the power input to drive the motor



nm = x 100
(18.5436) (0.746)
0.80 = 1
EP = 17.2919 kW
Solving for the gage pressure readings;
Applying energy equation from the water surface level of the source (A) to the suction
point of the pump (1), well have
ZA + VA2/2g + PA/w = Z1 + V12/2g + P1/w + HLA-1
Where; PA/w = 0 kPa, ( the water is open to the atmosphere)
VA = V1 (equal suction pipe diameter)
Z1 =0( used as the datum or reference line)

Thus; ZA = 1 +HLA-1
P1 = (ZA HLA-1) w = ( 5m-0.05m)(9.81 kN/m3) = 48.5595 kPa g
Applying energy equation from the discharge point of the pump (2) to the water surface
level of the reservoir(B) well have,
2
2 2
2
Z2 +2 +
= ZB + 2 + + HL2 B

Where: = 0 kPag(the water source is exposed to the atmosphere
V2 =VB (equal pipe diameter)
Z2 = 0 ( used as the datum or reference line)
2
Therefore:
= ZB + HL2-B
P2 = (ZB HL2-B ) w = (75 m-0.45 m) (9.81 kN/m3) = 731.3355 kPag
5. A pump discharges 150 litres per second of water to a height of 75 m. If the efficiency is 75%
and the speed of the pump is 1800 rpm, what is the torque in N-m to which the driveshaft is
subjected?
Solution:
Figure: 150

75m

Let ;
ep = pump efficiency
2
P= 60 =
= 9.81(0.150) (75)/(0.75) =147.15 kW, Thus;
2 2 1800
147.15 = 60 = 60
;
T= 0.781 kN-m = 781 N-m

Problem Solving:
1. A centrifugal pump delivers 227 m3/hr of water from a source 4 meters below the pump centre
line to a pressure tank whose pressure is 2.8 kg/cm2. Friction loss estimates are 2 meters in the
suction line and 1 metre in the discharge line. The diameter of the suction pipe is 250 mm and
the discharge pipe is 200 mm. Find:
a) The water horsepower
b) The kW rating of the driving motor if the pump efficiency is 70%
(Ans. a. 29 hp, b. 31 kW )
2. A pump is to deliver 80 galloons/min of water 600C with discharge pressure of 1000 kPag. Suction
pressure indicates 50 mmHg vacuum. The diameter of suction and discharge pipes are 5 inches
and 4 inches, respectively. If the pump has an efficiency of 70%. Determine the brake
horsepower of the pump. (Ans. 9.732 hp)
3. An acceptance test was conducted on a centrifugal pump having a suction pipe 25.4 cm in
diameter and a discharge pipe 12.7 cm in diameter. Flow was 186 m3/hr of clear cold water.
Pressure at suction was 114.3 mmHg vac and discharge pressure was 107 kPag at a point 91 cm
above the point where the suction pressure was measured. Input to the pump was 15 hp.
a) Determine the pump efficiency
b) If the pump runs at 1750 rpm, what net flow, head, and brake hp would be developed and
required if the pump speed were increased to 3500 rpm? Assume constant efficiency.
( Ans. a. 64.4%, b. 372 m3/hr, 56.8 m, 120 hp)
4. A motor driven pump draws water from an open reservoir A and lifts to an open reservoir B.
Suction and discharge pipes are 150 mm pipe and 100 mm inside diameter respectively. The loss
of head in the suction line is 3 times the velocity head in the 150 mm pipe and the loss of head
in the discharge line is 20 times the velocity head in the 100 mm pipeline.. Water level at
reservoir A is at elevation 6 meters and that of reservoir B at elevation 75 m. Pump centre line
is at elevation 2 m. Overall efficiency of the system is 73%. Discharge is 10 litres/sec. Determine
the following;
a. Power input of the motor
b. Reading in kPa of the pressure gauges installed just at the outlet and inlet of the pump.
(Ans. a. 9.51 kW, b. Po =732.34 kPag, Pi = 38.76 kPag )
5. A boiler feed pump receives 40 litres per second at 1800C. It operates against a total head of 900
meters with an efficiency of 60%. Determine:
a. The enthalpy leaving the pump in kJ/kg
b. Power output of the driving motor in kW
c. Discharge pressure in kPa if suction pressure of 4 MPa
(Ans. a. 773.57 kJ/kg, b. 523.3 kW, c. 11,850.3 kPa)
6. A plant has installed a single suction centrifugal pump with a discharge of 68 m 3/hr under 60 m
head and running at 1200 rpm. It is proposed to install another pump with double suction but
of the same type to operate at 30 m head and deliver 90 m3/hr
a. Determine the speed of the proposed pump.
b. What must be the impeller diameter of the proposed pump if the diameter of the existing
pump is 150 mm?
(Ans. a. 877 rpm, b. 145 mm )
7. A 40 m3/hr pump delivers water to a pressure tank. At the start, the gage reads 138 kPa until it
reads 276 kPa and then the pump will shut off. The volume of the tank is 160 litres. At 276 kPa
the water occupied 2/3 of the tank volume.
a) Determine the volume of water that can be taken out until the gage reads 138 kPa.
b) If 1 m3/hr of water is constantly used, in how many minutes from 138 kPa will the pump
run until the gage reads 276 kPa? Ans. a) 30.75 litres, b) 0. min.
8. A pump with a 400 mm diameter suction pipe and 350 mm diameter discharge pipe is to deliver
20,000 litres per minute of 15.60C water. Calculate the pump head in metres if suction gage is
7.5 cm below the pump centreline and reads 127 mmHg vacuum and the discharge gage is 45
cm above the pump centre line and reads 75 kPa. Ans. H=10.14 m
9. Water from an open reservoir A at 8m elevation is drawn by a motor driven pump to an open
reservoir B at 70 m elevation. The inside diameter of the suction pipe is 200 mm and 150 mm
for the discharge pipe. The suction line has a loss of head 3 times that of the velocity head in the
in the 200 mm pipe. The discharge line has a loss of head 20 times that of the head in the
discharge pipeline. The pump centreline is at 4 m. Overall efficiency of the system is 78% For a
discharge rate of 10 li/s, find the power input to the motor and the pressure gages reading
installed just at the outlet and inlet of the pump in kPag. Ans. P input = 7.825 kW, Ps= 39.14
kPa, Pd= 650.80 kPa
10. The rate of flow of water in a pump installation is 60.6 kg/s. The intake static gage is 1.22 m
below the pump centreline and reads 68.95 kPa gage;the discharge static gage is 0.61 m below
the pump centre line and reads 344.75 kPagage. The gages are located close to the pump as
much as possible. The area of the intake and discharge pipes are; 0.093 m2 and 0.069 m2
respectively. The pump efficiency is 74%. Take density of water equals 1000 kg/m 3. What is the
hydraulic power in kW. Ans. P=17.09 kW
11. A pump delivers 20 cfm of water having a density of 62 lb/ft3. The suction and discharge gage
reads 5 in.Hg vacuum and 30 psi respectively. The discharge gage is 5 ft above the suction gage.
If pump efficiency is 70%, what is the motor power? Ans. P=4.31 hp
12. In a test of a centrifugal pump driven by an electric motor, the suction pipe is 10 in. in diameter
and its gage indicates a partial vacuum of 2.5 ft of water. The discharge pipe is 5 in. in diameter,
is 2 ft. higher than the suction gage and shows pressure of 50 ft of water. If the pump is
discharging 1.6 ft3/s and the electrical power input is 12 kW, what pump efficiency is indicated
assuming motor efficiency of 85%. Ans. 74.04%
13. Oil is being pumped from a truck to a tank 10 ft higher than the truck through a 2 in. galvanized
pipeline 100 ft long. If the pressure of the discharge side of the pump is 15 psi, at what rate in
gpm is oil flowing through the pipe?. The oil has an specific gravity of 0.92 at the temperature
in the pipe. Ans. 542.44 gal/min
14. A centrifugal pump delivers 80 litres per second of water on test suction gage reads 10mmHg
vacuum and 1.2 m below pump centreline. Power input is 70 kW. Find the total dynamic head
in metres. Use np (pump efficiency) = 74%. Ans. H=66m

Characteristics of Centrifugal Pump


1. Specific Speed is the speed at geometrically similar impeller of a pump would run to
discharge 1 gpm at 1 foot head. Specific speed of centrifugal pump impeller is the rotative
speed at which a geometrically similar impeller would run if it were of such size as to raise
75 kg of water per second against one metre head.

Ns = 3 ; rpm, in English System
4
Where;
Ns = specific speed in rpm
N= rotational speed, in rpm
Q= discharge, in gpm
H= head, in feet
0.0149
Ns = 3 rpm, in Metric system
4
Where:
N= pump shaft speed in rpm
H , or TDH =head in metre per stage
Q = is the discharge in litres/min
Note; For double suction pumps Q is divided by two and for multi-stage pumps, H is divided
by the number of stages. In Figure1.13 shows the relationship of pump efficiency, specific
speed, geometry of pump and capacity. As the specific speed of the pump increases, the
ratio of impeller outer diameter, D1, to the impeller centre diameter, D2 , decreases. The
ratio becomes 1.0 for a propeller type impeller.
The observation from the Figure that low specific speed can be designated as low capacity,
that means the head is developed mainly through centrifugal force and less of axial force.
Also high specific speed can be designated as high capacity, which indicates that head is
developed mainly through axial force and less of centrifugal force.
By using Figure 1.14 Pump specific speed when N=3500 rpm, H=120 ft and Q=1200 gpm,
the Ns =3350 rpm. It is important to note that specific speed is always calculated at the Best
Efficiency Point (BEP) of the pump at maximum impeller diameter.

Example Problem; A double suction, single stage centrifugal pump delivers 4000 gpm of water from a well where
the water level varies from 2.5 metres from high tide to low tide. The pump centre line is measured to be 3 m
above the water level at high tide. The pump discharges the water to an open surface condenser located 4.5 m
above the pump centreline. The head loss due to friction in the suction and discharge pipe is 1 m and 2.5 m,
respectively. The pump is directly coupled in a motor with 1800 rpm. Determine:

a) Total suction head, m


b) Total discharge head, m
c) Pump specific speed
d) Impeller diameter in mm

Solution:

FIGURE;

Solving for the total suction head

Total suction head= 2.5 + 3 + 1 = 6.5 m

Total discharge head = 4.5 + 2.5 = 7.0 m


Solving for pump specific speed
3.28
Ns = , where, H=13.5 m ( )= 44.28 ft
43 1

4000
1800
2
Ns = 3 =4689.5567 rpm, Using Figure 1.13, the specific speed suggests a mixed flow type of
(44.28 )4
impeller.

Solving for the pump impeller diameter,


()
V= 60
= 60

V=2 = 2(9.81)(13.5) =16.2748 m/s

Substituting:
(1800)
16.2748 m/s = 60

D= 0.1727 m = 172.6810 mm

Example 02]

Calculate the impeller diameter of the centrifugal pump that requires 15 m head to deliver water if pump
speed is 1500 rpm.

Solution

V=

Solving for velocity

V=2 =2(9.81)15 = 17.15 m/s

Thus;

17.15 = D( 1500/60)

D= 0.21843 m = 218.43 mm

Characteristic Curve and Best Efficiency Point

Pump performance curves are graphical representation of the characteristic curve of a certain pump model
which shows the pump capacity ,(gpm), impeller diameter (inches), total dynamic head, (feet) brake
horsepower input (bhp) and pump efficiency in percent. A typical pump performance curve is shown below:
The highest point on the efficiency curve is the (Best efficiency Point)

FIGURE:

H-Q Characteristics and Efficiency

Example;

Required; Head, horsepower, and efficiency at 160 gpm,

Solution ; From Figure, bhp = 7bhp; head =130 ft, best efficiency is at 160 gpm

NET POSITIVE SUCTION HEAD


Pumps are designed to operate continuously for long period of time. To achieve this, the pump utilizes the
fluid that it is pumping for its lubrication and cooling. Damage in the pump may occur when circulation of
liquid stops for long period of time while the pump is operating.

Damage can also occur when the pressure at any point inside the pump drops below the vapour pressure
corresponding to the temperature of the liquid. Because at this condition, the liquid will vaporize and
eventually forms vapour bubbles as it enters the inlet of the pump. This vapour bubbles then collapse or
implodes at the surface of the impeller creating tremendous physical shock to the edges of the impeller. This
process of vapour formation is cavitation. Another undesirable result of cavitation is adverse noise
accompanied by heavy vibrations.

Since cavitation occurs mainly at the inlet of the pump, we can consequently say that it is related to the
pump suction head conditions. Net positive Suction Head, NPSH, is an index where the pump may operate
without cavitation. NPSH is the difference between actual suction pressure and saturation vapour pressure
of the liquid.

Two kinds of NPSH,

1.0) NPSHr
2.0) NPSHa

NPSHr for a particular pump is experimentally determined and provided by the manufacturer and is a
function of pump design. NPSHa is determined by plant engineer during the design and proposed installation
of the pump and is a function of the system where the pump will operate. To avoid cavitation, it is necessary
that NPSHa is equal or greater than NPSHr.

To avoid cavitation, NPSHa>=NPSHr

If NPSHa is less than or equal to NPSHr

1.0) Decrease the suction lift by changing the plant layout and raising the source on which the pump draws
water
2.0) Reduce the suction head by using pumps with larger capacity but operating it in partial loads or
speeds.

Cavitation- is defined as the formation of cavities of water vapour in the suction side of a pump due to low
suction pressure.

Causes of Cavitation

1.0) Low suction pressure


2.0) Low atmospheric pressure
3.0) High liquid temperature
4.0) High velocity
5.0) Rough surface and edges
6.0) Sharp bends

Bad Effects of Cavitation


1.0) Drop in capacity and efficiency
2.0) Noise and vibration
3.0) Corrosion and pitting

2.0) Similar Pumps

1 1 2 2
a.) 3 = 3
14 2 4
1 2
b.) = ; where; D= impeller diameter
1 13 1 23
3.0 Same Pump (Affinity Laws)
Pump affinity laws are rules that express the relationship of pump capacity head, and bhp when the
speed or impeller diameter is changed. Assuming that the efficiency is the same for both conditions.
a) Constant impeller diameter, variable speed
1
2
= 1
2
1
2
= [1 ]2
2

1
2
=[1 ]3 P= Power
2
b) Constant speed, variable impeller diameter
1 1 1
2
= 1 ; 2
= [1 ]2 2
= [1 ]3
2 2 2

Similarity Laws of Pumps

1. Q is directly proportional to D3N


2 3
1
= 23 x 2
1 1
2. H is directly proportional to N2
2
1
=( 2 )2 x (2 )
1 1

3. 3D5
2
1
=( 2 )3(2 )5
1 1

Example Problem:

1.0) A pump delivers 500 gpm against a total head of 200 ft and operating at 1770 rpm. Changes have
increased the total head to 375 ft. At what rpm should the pump be operated to achieve the new head at
the same efficiency.
Solution:

= [ ]2 ;

2

=[
] ; N2 = 2423.67 rpm

2.0) It is desired to deliver 5 gpm at a head of 640 ft in a single stage pump having a specific speed not to exceed
40. If the speed not to exceed 1352 rpm, how many stages are required?
Solution:
13525
Ns = 3 = 40 = 640 ; n= 2 stages
4 ( ) /

3.0) The power output is 30 hp to a centrifugal pump that is discharging 900 gpm and which operates at 1800
rpm against a head H= 120 ft, 220V, 3 phase, 60 hertz. If this pump is modified to operate 1200 rpm,
assuming its efficiency remains constant, determine its discharge in gpm, the theoretical head it imparts to
the liquid and the power input to the pump.
Solution:
1 1
=
2 2
900 1800
=[ ] ; Q2 = 600 gpm. , 2
2 1200
1
2
= [1 ]2
2
120 1800
2
= [1200]2
= = 53.33 ft
, P2

= [ ]3


= []3

P2 = 8.89 hp

Problem Solving:
1.0) A pump operating at 1750 rpm delivering 500 gpm against a total head of 150 ft. Changes in the piping
system have increased the total head to 360 ft. At what rpm should the pump be operated to achieve
this new head at the same efficiency? Ans. 2711.09 rpm
2.0) A DC driven pump running at 100 rpm delivers 30 litres per second of water at 400C against a total
pumping head of 27 m with a pump efficiency of 60%. Barometer pressure is 758 mmHg. What pump
speed and capacity, would result if pump rpm were increase to produce a pumping head of 36 m
assuming no change in efficiency. Ans, 115.47 rpm, 34.64 L/s
3.0) A centrifugal pump discharge 20 L/s against a head of 17 m when the speed is 1500 rpm. The diameter
of the impeller was 30 cm and the brake horsepower was 6.0. A geometrically similar pump 40 cm in
diameter is to run at 1750 rpm, Assuming equal efficiencies, what brake horsepower is required? Ans.
Bhp = 40.14 bhp
4.0) A two stage centrifugal pump delivers 15,000 kg/hr of 1100C water against 76 m head at 3500 rpm. What
is the specific speed of the pump? Ans. Ns = 780.39 rpm
5.0) A pump running at 1000 rpm delivers water against a head of 300 m. If the pump speed will increased
to 1500 rpm, what is the change in head? Ans. H= 375 m
6.0) A test on a centrifugal pump operating at 1150 rpm showed a total head of 37.6 ft at a capacity of 800
gpm. Estimate the total head and capacity if the pump were operated at 1750 rpm. Assume normal
operation at point of maximum efficiency in each case. Ans. H2 = 87.07 ft. , Q2 =1217.4 gpm
7.0) A double- suction centrifugal pump delivers 3 m3/s of water at a head of 15 m and running at 1200 rpm.
Calculate the specific speed of the pump. Ans. Ns = 9958.56 rpm
8.0) Determine the performance of a centrifugal pump with initial flow of 150 gpm, at an initial head of 120
ft, and initial power input of 5 bhp. The impeller diameter is changed from 9.5 inches to 8.0 inches. Ans.
Q2 = 126.3158 gpm; H2 =85.0970 ft. ; BHP2 =2.9859 bhp
9.0) Determine the performance of a centrifugal pump with initial flow of 150 gpm, at an initial head of 120
ft, initial power input of 5 bhp. The speed is changed from1700 rpm to 3000 rpm. Ans. Q 2 =264.7059
gpm, H2 = 373.7024 ft, BHP2 = 27.4781 bhp
10.0) A centrifugal pump operating at 1150 rpm showed a total head of 40 ft at a capacity of 600 gpm.
The impeller diameter is 10.5 in. Estimate the total head and capacity of a geometrically similar pump
at 1150 rpm with an impeller diameter of 10 inches. Ans. H2= 36.28 ft, Q2 =518.3 gpm
11.0) A double suction, single stage, centrifugal pump delivers 900 m3/hr of sea water(S.G. =1.03)
from a source where the water level varies 2 m from high tide to low tide level. The pump centreline is
located 2.6 m above the surface at high tide level. The pump discharges into a surface condenser, 3 m
above the pump centreline. Loss of head due to friction in the suction pipe is 0.80 m and that in the
discharge side is 3 m. Pump is directly coupled to a 11750 rpm, 460 Volts, 3 phase, 60 hertz motor.
Calculate the specific speed of pump in rpm Ans. Ns = 5149.20 rpm . Ns =5149.20 rpm.

Net Positive Suction Head Available, NPSHa


Formula:
NPSHa= hp( + or -) hSL -hV - HL
Where:
Hp = absolute pressure head on the surface of the liquid source in meters. This will be the atmospheric
pressure corresponding to its altitude when the liquid surface is open.
HLS = the height of the liquid surface from the pump centreline , designated as positive when suction
head and negative when suction lift, in meters.
HV = head corresponding to the vapour pressure of the liquid at liquid temperature, can be determined
using steam tables, in meters
HL= head loss due to friction and turbulence, in meters.

Example Problems:
1.0) Determine the NPSH available with following water conditions: Water from a well with a
temperature 270C at sea level, with head loss of 0.45m and a suction lift of 3 m.
Solution;
Figure;
NPSHa = hp + or hSL hv - HL
From Steam tables: specific volume of water at 270C is; vf = 1.0035 x 10-3m3/kg and the specific
weight is ; = 9.7758 kN/m3
Since the water source is open to the atmosphere, hp=101.325 kPa
101.325
Hp = = 9.7758 / = 10.3649 m
.
HSL = -3 m ( since it is suction lift)
The vapour pressure of water at 270C is 3.567 kPa
3.567
Hv = = = 0.3649 m
9.7758
.
HL = 0.45 m
Thus;
NPSHa= (10.3649-3-0.3649-0.45) = 6.55 m

Example Problems on NPSH

2.0)A closed tank contains liquefied butane gas whose specific gravity is 0.60. The tank pressure is 1.6
MPa gage which is also the equilibrium vapour pressure of butane at the pumping temperature. Suction
line losses is 1.5 m of gas and the static elevation gain is 4 m. What is the net positive suction head
available (NPSH)?

Solution:
Use (+) for static elevation gain
+
NPSH = 9.81()
+ S Hf
= 0 + 4 -1.5 = 2.5 m available
Problem Solving:
1.0) A condensate pump draws water from a condenser which maintains a pressure head of 724
mmHg vacuum. The friction losses at the piping system between the pump and the condenser
is measured to be 1.5 m. Determine the minimum height of water in the condenser that needs
to be maintained if the NPSH r for the pump is 3.5 m. Ans. hLS = 4.7 m.
2.0) A condensate pump at sea level take water from a surface condenser where the vacuum is 15
in.Hg. The friction and turbulence in the piping in the condenser hot well and the pump suction
flange is assumed to be 6.5 ft. If the condensate pump to be installed has a required head of 9
ft, what would be the minimum height of water level in the hot well that must be maintained
above the centreline of the pump to avoid cavitation? Ans. S= 15.5 ft= hLS
3.0) A dearator heater supplies 150 l/min of dearated feed water into a booster pump at 115 0C
pumping temperature. The heater pressure is maintained at 100 kPag by bled steam. Pump
centreline is located 1 m above the floor level. Suction line losses is 0.60 m. Determine the
minimum height of water level in the heater that must be maintained above the centreline of
the pump to avoid cavitation, if the pump to be installed has a required suction head of 5.8 m.
Ans. S= 3.25 m
Pumps in Parallel ( to increase discharge at the same head)
FIGURE:

PUMPS in SERIES
(to increase head with same discharge}
FIGURE

PUMP SELECTION
The following are important items to consider in selecting a pump.
1.0) The desired flow rate
2.0) The suction lift available (NPSHa)
3.0) The total dynamic head in which the pump will operate, TDH
4.0) Nature of liquid handled
5.0) Suction conditions
6.0) Type of drive, motor or engine
7.0) Type of service, continuous or intermittent
8.0) Number of units
9.0) Climate conditions
RECIPROCATING PUMP

Reciprocating Pump is a positive displacement Pump wherein the pumping action is accomplished by
forward and backward movement of a piston or plunger inside a cylinder usually provided with valves.

A POSITIVE DISPLACEMENT PUMP makes a fluid move by trapping a fixed amount and forcing (displacing) that
trapped volume into the discharge pipe.

Some positive displacement pumps use an expanding cavity on the suction side and a decreasing cavity on the
discharge side. Liquid flows into the pump as the cavity on the suction side expands and the liquid flows out of
the discharge as the cavity collapses. The volume is constant through each cycle of operation. They are also
called constant flow machines. A positive displacement pump must not operate against a closed valve on the
discharge side of the pump, because it has no shutoff head like centrifugal pump. A positive displacement pump
operating against a closed discharge valve continues to produce flow and the pressure in the discharge line
increases until the line bursts, the pump is severely damaged, or both. A relief or safety valve on the discharge
side of the positive displacement pump is necessary.

Piston types are used for low pressures, light duty or intermittent service. Less expensive than plunger design
but cannot handled gritty liquids.

Plunger types are used for high pressure, heavy duty or continuous service. Suitable for gritty and foreign
material service and more expensive than the piston design.

FIGURE:

Types of Reciprocating Pumps

1. Direct acting steam pump__ This type has a steam cylinder and a common piston rod. As there is no
lap, the steam is admitted throughout the length of the stroke, hence the pressure volume diagram of
the steam end is a rectangle. Consequently the water end flow diagram will also be a rectangle. With
the discharge flow constant throughout the length of the stroke and going down to zero value at the
instant or reverse at the end of each stroke
2. Cam and flywheel Reciprocating Pump_ this type is driven by cross compound or triple expansion
steam engines.
3. Power driven Pump_ this type receives its forward and backward motion of the piston and plunger by
means of a crank and connecting rod.

Note: Reciprocating pump can be single acting or double acting, simplex, duplex, triplex depending on the
number of water cylinder on the machine. Most built in double acting.

Pump Material

The pump material can be Stainless Steel (SS 316 or SS 3040, CAST IRON ETC. It depends on the
specification of the pump. In water industry and for pharma applications SS 316 is normally used, as
stainless steel gives better results at high temperatures.

Parts of Reciprocating Pump

1. An air chamber- is a metal box installed in the discharge side of the pump in which air is maintained to
cushion the flow of the water from the discharge of the pump as that the final flow will be more or less
continuous.
2. Air pressure valve- this should be installed on the discharge side between the pump and any other
valve.
3. Foot valve and strainerThis should be installed at the end of suction pipe. Foot valve should be of a
size at least equal to the size of the suction pipe.. The area of the strainer should be at least 3 times
the area of the suction pipe in order to minimize head loss at this point.

Characteristics of Reciprocating Pump


1. Piston displacement
If piston rod neglected

Vd= 2( 4 ) D2LN
If piston rod considered

Vd= 4 D2LN + 4 (D2-d2 ) LN; where; d = diameter of piston rod
2. Q = AV
3. Slip = Vd- Q

% Slip =[
] x 100

4. Volumetric efficiency = = v = 1 + c c(1)
2

Example Problem

A 25 x 15 x 25 cm direct- acting duplex pump with 4 cm piston rods makes 35 double stroke per cylinder per
minute and delivers 510 litres per minute of 380C water against a total head of 140 metres. The steam is
supplied to the steam end at a pressure of 1034 kPa gage and exhaust to the atmosphere. Amount of steam
supplied is 1725 kg/hr @ 0.98 quality. Barometric reading of 752 mm Hg. Assume that the piston makes full
stroke; find:

a. Piston displacement of the pump in cm3/hr


b. Pump slip
c. Water power delivered
d. Pump overall efficiency

Solution:

Figure:

a).Solving for the piston displacement of the pump

Piston displacement = displacement at the head end + displacement at crank end.

Vd= VHE + VCE



Vd= 4 D2LN + 4 ( D2-d2) LN = 4 LN [ D2 + ( D2 d2)

Where;

L= 25 cm = 0.25 m
60
N= 35 x 2 = 70 x = 4200 cycles/hr

D= 15 cm = 0.15 m

d = 4 cm = 0-.04 m

Vd= 4 (0.25) (4200) [(0.15)2+ (0.15)2- (0.04)2] = 35.79 m3/hr

Solving for the pump slip



Vol. efficiency = x 100%

Q= 510 litres/min = 0.510 m3/min


3 3
Vd = 35.79 = 0.5965
0.510
Vol. efficiency = = 0.85498 x 100% = 85.498%
0.5965

Pump slip = 100%- 85.498% = 14.502%

Solving for power water delivered

Fluid Power = QH

of water at 380C = 992.95 kg/m3 = 9.7375 kN/m3

Q= 0.510 m3/min = 0.0085 m3/s

H= 140 m

FP = 9.7375 (0 0085) (140) = 11.59 kW

Solving for the pump Thermal efficiency



et = =
( 1 2)

ms = 1725 kg/hr = 0.479167 kg/s


1 101.325
P1= 1034 kPa + 752mmHg(760 ) ( 1
) = 1134.2258 kPa

From steam Tables

h1= hf + x hfg = 787.393 + 0.98 ( 1995.4326) = 2742.9162 kJ/kg

hf2 @ atm. Pressure

atm. Pressure = 752 mm Hg = 0.00258 MPa

hf2 = 417.7582 kJ/kg


11.59
et = 0.479167 ( 2742.9162417.7582) = 1.043%

Rotary pump- is a positive displacement pump consisting of a fixed casing containing gears, cams, screws,
vanes, plungers or similar elements actuated by rotation of the drive shaft.

Types of Rotary pumps

1) Cam and piston pump- type of rotary pump consist an eccentrically bored cam rotated by a shaft
concentric cylindrically bored casing, with an aburment or follower so arranged that with each rotation
of the driveshaft a positive quantity of liquid is displaced from the space between the cam and the
pump casing.
2) Screw pumpis a type of rotary pump consists of two or three screw motors so arranged that as the
rotors turn liquid fills the shape between the screw threads and is displaced axially as the rotor threads
mesh.
3) Vane pump- a type of rotary pump consist of one rotor in a casing machined eccentrically to the drive
shaft. The rotor is fitted with a series of vanes, blades or buckets which follow the bore of the casing
thereby displacing the liquid with each revolution of the driveshaft.
4) Gear pump- type of rotary pump, consists two or more gears, operating closely fitted casing so
arranged that when the gear teeth unmesh on one side liquid fills the space between the gear teeth
and is carried around in the tooth space to the opposite side and displaced as the teeth mesh again.

Special Classification of Pump based on suction lift


1. Shallow well pump(ordinary centrifugal pump for suction lift up to 25 ft)
2. Deep well Pump___ (centrifugal pump with injector for suction lift up to 120 ft)
3. Turbine pump (multi stage pump, for suction lift up to 300 ft)
4. Submersible pump (multi stage pump driven by submersible motor)
Deep well Pumps may divided into:
1. Plunger or reciprocating
2. Turbine
3. Ejector centrifugal type
4. Air lifts
Plunger types-a ball valve, plunger and check valves are used in this pump. In operation only the
plunger moves. When the plunger is raised a vacuum is created below it, a water flows through the
check valve in the plunger to be lifted on the next upward stroke of the plunger.

Turbine type Pumps- These pumps represents the application of vertical centrifugal pump to dep
well service and are built for heads up to 305 metres and capacities up to26295 litres/min.
Ejector centrifugal pump- type of deep well pump used for small capacities combines a single stage
centrifugal pump at the top of the well and an ejector o jet located down in the water This is best
suited when the lifts is 7.6 m or over and capacities up to 190 liters/min net discharge
Air lifts- another method of pumping wells is by air lift with compressed air being admitted to the
well to lift water to the surface.
CHAPTER 3
COMPRESSORS
Compressors are machines that are used to transfer another gasses from one location to another at a
higher differential pressure ranges. There are various types of compressors and they are classified in two major
categories: (1) dynamic or also known as continuous flow or turbo compressor and (2) positive displacement or
intermittent compressor.

Dynamic or continuous flow are types of compressor in which the flow of gas is accelerated thru high
speed rotating element, converting velocity head into pressure head. Continuous flow includes centrifugal and
axial flow compressors.

Positive displacement or intermittent flow type compressor, however, are those units that confine gas
in a closed space, reducing its volume and discharging it at a higher pressure. Intermittent flow includes
reciprocating, screw, and root types of compressors.

Some other ways of classifying compressors includes the following:

a) Number of stages: single, two, three-stages or multi-stage


b) Reciprocating compressor element: single acting or double acting
c) Cylinder arrangement: vertical, horizontal, V-type
d) Cooling system: water cooled, air cooled
e) Mounting condition: portable, stationary

RECIPROCATING COMPRESSORS
Reciprocating compressors are positive displacement, intermittent flow machines that are capable of
delivering air from a pressure of 35 psig up to 250 psig. They are widely used in industrial applications because
of their overall efficiency, wide range of capacity simplicity of use and compactness.

Some of the disadvantages of reciprocating compressors are the excessive vibrations due to the forces
exhibited by the reciprocating movement of the piston. This requires strong machine foundation and higher
maintenance cost compared with other types of compressor. Figure 3.1 shows a typical reciprocating
compressor with its parts.

FIGURE 3.1: Portable Air Compressor

A reciprocating compressor resembles automotive engines and can be air-cooled or liquid-cooled.


Generally, low capacity compressor are air-cooled and they have air fins cast around the cylinder to dissipate
heat. On the other hand, liquid-cooled compressors have cooling fins and water jackets cast around the
cylinder to circulate water.

Reciprocating compressors needs lubrication to reduce wear, provide cooling and to act as a sealant
between moving parts. Lubricant is distributed thru a splash system crank and connecting rods with oil in the
crankcase.
Non-lubricating reciprocating compressors use special design piston and non-metallic piston rings
without oil in the crankcase. They have higher maintenance cost because valves and piston rings wear faster
than in lubricated reciprocating compressor.

FIGURE 3.2: Schematic view of air-cooled single-stage, single-acting reciprocating compressor


Single-acting Reciprocating Compressor
The cylinders of reciprocating compressors can be classified as single-acting and double acting.

Single-acting compressors as shown in Figure 3.3, compresses gas in one side of the piston only. They
are commonly air cooled and widely used for small capacity applications.

FIGURE 3.3: Single-acting Reciprocating Air compressor and P-v diagram

Compression stroke starts at point 1. As the compressor compress the air from point 1 to 2, the suction
valves immediately closed, shutting off the suction line of the cylinder. At point 2, the discharge valves opens
and compressed air is pushed out of the cylinder into the discharge line of the cylinder.
The discharge stroke is completed at point 3. At this point, the piston returns to point 4, where the
pressure drops and closing the discharge valve. The clearance between the end of the cylinder and the top dead
center (TDC) is known as the clearance volume.
The piston expands from point 3 to 4. And the intake stroke occurs from point 4 to 1, which is also known
as the compressor capacity. The complete cycle is shown in the P-v diagram in figure 3.3.

Piston Speed, v
Piston speed is measured as the total distance travel by the piston multiplied by the angular
speed.
N
v = 2L 60

Where: v = piston speed, m/s


L = length of stroke, m
N = compressor angular speed, rpm

Volume Displacement, VD
VD = V1 V3
N
VD = AL 60
D2 LN
VD = ( ) (No. of cylinder) (No. of piston action)
4 60
Where: D = piston diameter, m
L = stroke, m
N = compressor angular speed, rpm
No. of piston action = 1 for single-acting; 2 for double-acting

Double-acting Reciprocating Compressor


Double-acting compressors as shown in the figure 3.4, compresses gas in both sides of the piston. For
the same motor speed, double-acting compressors deliver twice of that single-acting compressors. They are
water-cooled and mostly used in large capacity applications.

FIGURE 3.4: Schematic view of double-acting reciprocating compressor

The P-v diagram of a double-acting reciprocating compressor is shown in Figure 3.5. The cycle occurs at
the opposite side of the piston.

FIGURE 3.5: Double-acting Reciprocating Air compressor and P-v diagram

COMPRESSOR FUNDAMENTALS
Compressor Work, Wc
a) Work for Polytropic Compression
n1
nP1 V1 P2 n
Wc = [( ) 1]
n 1 P1

b) Work for Isentropic Compression


k1
kP1 V1 P2 k
Wc = [( ) 1]
k 1 P1

c) Work for Isothermal Compression


P2
Wc = P1 V1 ln ( )
P1
Note: Pressure must be in absolute values
Compressor Capacity, V1
Compressor capacity is the actual volume of air drawn in by the compressor as measured at the intake
pressure and temperature.
V1 = V1 V4
ma RT1
V1 = P1

Where: R = ideal gas constant,


Rair = 0.287 KJ/kg-K for air
PVT Relationship




= () = ()

Where: 1 n 1.4 = polytropic compression


n = k = 1.4 = isentropic compression
n = 1 =isothermal compression
Volumetric Efficiency, v
Volumetric efficiency describes how efficient air is being drawn into the cylinder of the air-
compressor. It is the ratio of the amount of air drawn into the cylinder of the air-compressor. It is the
ratio of the amount of air drawn in divided by the volume displacement or the maximum possible
amount of air can be drawn in.
actual volume
v = volume displacement 100%

V1 V1 V4
v = VD
100% = VD
100%

And; V1 = V3 + VD = cVD + VD
Where: c = percent clearance ranges 3% to 10%
n1
V n1 P n
(V4 ) = (P3 ) ; P3 = P2, P4 = P1
3 4

1 1
P n P n
V4 = V3 (P2 ) = cVD (P2 )
1 1

Substituting;
1
P
V1 (cVD +VD )cVD ( 2 )n
P1
v = VD
100% VD
100%

Therefore;
V1
v = VD
100%

1
P n
v = 1 + c c (P2 ) 100%
1

It is evident that as the clearance becomes smaller, the volumetric efficiency increases.
Compressor Efficiency
The compressor efficiency is the ratio of the compressor power output over the brake power input to
the compressor. Consider the compressor shown in Figure 3.6:
FIGURE 3.6: The compressor as the system
Wc
c = BP
100%

Example:
A single-acting reciprocating air compressor with a clearance of 5% receives air at 100 kPa and 30C and
is delivered at 450 kPa. The bore and stroke are 350 mm and 390 mm, respectively when operating at 1000 rpm.
Determine:
a) Volume displacement, in m3/s
b) Volumetric efficiency, in percent
c) Compressor capacity, in m3/s
d) Compressor work, in kW
e) Compressor efficiency, in percent, if the brake power input to the compressor is 120 kW.
f) Temperature of air at the discharge, in C
Assume no pressure drop in the intake and discharge port of compressor and take the compression
and expansion process to be PV1.3 = C.

Solving for the volume displacement, VD


D2 LN
VD = ( ) (No. of cylinder) (No. of piston action)
4 60

(0.350 m)2 1000 rpm


VD = (0.390 m) ( ) (1)(1)
4 60


= .

Solving for the volumetric efficiency, v


1
P n
v = 1 + c c (P2 ) 100%
1

1
450 kPa 1.3
v = 1 + 0.05 0.05 (100 kPa) 100%

= . %
Solving for the compressor capacity, V1
V1
v = VD
100%

V1
0.8910 = m3
0.6254
s


= .

Solving for the compressor work, in kW


n1
nP1 V1 P2 n
Wc = [( ) 1]
n1 P1

m3 1.31
1.3(100 kPa)(0.5572 ) 450 kPa
s 1.3
Wc = 1.31
[(100 kPa) 1]

= .
Solving for the compressor efficiency,
Wc
c = 100%
BP
100.1912 kW
c = 120 kW
100%

= . %
Solving for the temperature of air at the discharge, in C



= ()

.
.
(+)
= ( )

T2 = 428.7301 K
= .

Example:
A double acting compressor with a volume displacement of 0.432 m3/s, delivers air at 725 kPa at a rate
of 0.188 m3/s. The inlet condition of air 100 kPa and 30C and the angular speed of the compressor is 200 rpm.
For a compression and expansion process given by PV1.3 = C, determine:
a) The percent clearance of the compressor
b) The bore and stroke, in meters, assuming that the stroke is equal with the bore and the
volume displacement of the crank end and head end are the same.
Solving for the percent clearance of the compressor;
m3
V1 0.188
s
v = VD
100% = m3
100%
0.432
s

v = 43.52%
Then;
1
P n
v = 1 + c c (P2 ) 100%
1

1
725 kPa 1.3
0.4532 = 1 + c c (100 kPa)

= . %
Solving for the bore and stroke, in mm;
D2
VD = ( 4
) LN (No. of cylinder) (No. of piston action)

m3 (D)2 200 rpm


0.432 = (D) ( ) (1)(2)
s 4 60
= .
= .

Actual Compressor Cycle


An increase in area occurs in actual compressor cycle because of the fluid losses thru the inlet and
discharge ports of the compressor which cause pressure drops in the cycle. Other contributors to the pressure
drop in the compressor are the friction losses and fluid slippage past the piston rings, intake and discharge valve.
A larger area covered by the cycle in the P-v diagram, as shown in Figure 3.7, means larger horsepower input
required to the compressor.

FIGURE 3.7: Actual Compressor P-v diagram as given by an indicator card

Example:
A single acting reciprocating compressor receives air at 100 kPa and 30C and delivered at 700 kPa. The
clearance volume is taken to be 10% and the compression and expansion process to have a polytropic exponent
of 1.3. The compressor piston displacement is 450 cm3 and operating at a speed of 850 rpm. Determine the mass
of air compressed, in kg/hr and the required compressor power, in kilowatts when the pressure drop at the
suction and discharge port are 10 kPa and 15 kPa, respectively.
Solving for the mass of air compressed, in kg/hr
P1 V1 = ma RT1
P1 V1
ma = RT1

Solving for V1, form volumetric efficiency, v;


1
V1 P n
v = = 1 + c c ( 2)
VD P1

Solving for the volume displacement, VD, im m3/s


N 850rpm
VD = AL 60 = (4.5 104 m3 ) ( 60
)

m3
VD = 6.375 103 s

Substituting to solve for the actual volume of air taken in by the compressor, V1;
1
V1 715 kPa 1.3
m3
= 1 + 0.10 0.10 ( 90kPa )
(6.375103 )
s

V1 =
(3.8732
m3 3600 s
103 s
) ( 1 hr ) =
m3
13.9434 hr

For mass of air;


ma =
m3
(90kPa)(13.9434 )
hr
kJ
(0.287 )(30+273)K
kgK

kg
ma = 14.4307 hr

The mass taken in by the compressor was reduced by 10% because of the pressure drop at the suction
and discharge port of the compressor.
Solving for the compressor power required, in kW
n1
nP1 V1 P2 n
Wc = [( ) 1]
n 1 P1

m3 1.31
(1.3)(90kPa)(3.8732 103 ) 715 kPa
s 1.3
Wc = 1.31
[( 90kPa ) 1]

= .
In general, single compressors are generally used for pressures from 35 to 100 psig, and a two-stage
compressor are used from 100 to 250 psig.
Multi-stage Reciprocating Compressor
Two-stage compressor
In multistage compressor, the gas is delivered by the low pressure cylinder at intercooler pressure to
succeeding stages. In this way, the low pressure cylinder determines the volumetric efficiency of the whole
machine because whatever the low pressure cylinder passes to the succeeding stages must be discharged.
Figure 3.8 shows the P-v diagram of a two-stage compressor.
FIGURE 3.8: Two-stage reciprocating compressor

By using multistage compressor, the power input to the compressor is lessen, the gas discharge temperature and
pressure differential are decreased. Intermediate pressure, PHX, for two-stage compressor can be theoretically
approximated by:

PHX = P1 P4
Where: P1 = pressure intake at the first stage
P4 = pressure at the second stage discharge
The compressor work for two-stage, WC12 is given by;
n1
2nP1 V1 PHX n
WC12 = n1
[( P ) 1]
1

The heat rejected by the intercooler;


Q HX = ma Cp (T1 THX )

Three-stage compressor
Figure 3.9 shows the P-v diagram of a three-stage compressor.

FIGURE 3.9: Three-stage reciprocating compressor

For two stage compressor, intermediate pressure, PHX and PHY, can be theoretically approximated by:
3
PHX = P1 2 P6
3
PHY = P1 P6 2

Where: P1 = pressure intake at the first stage


P6 = pressure at the third stage discharge

The compressor work for three-stage, WC123 is given by;


n1
3nP1 V1 PHX n
WC123 = n1
[( P ) 1]
1

For ideal conditions:


PHX P P
P1
= PHY = P 6
HX HY

Example:
A single acting, two stage, reciprocating compressor receives air at 100 kPa and air capacity of 0.05 m 3
per second and delivered at 400 kPa. The clearance volume is taken to be 8%. The compression and expansion
process is isentropic with compressor mean piston speed of 150 m/min. Assuming that each piston has the same
amount of stroke, no pressure drops at each suction and discharge ports of the compressor and perfect
intercooling, determine:
a) The piston diameter, in cm
b) The total power required, in kW
c) The heat loss at the intercooler, in kW

a) Solving for the diameter of the piston at the first stage, d1;

VD1 = ALN = ( d1 2 ) LN
4
Piston speed: V = 2 LN
150 m
60 s
= 2 LN
m
LN = 1.25 s

Solving for the volume displacement, VD1 at the first stage;


1
V1 P k
= VD1
=1+c c ( PHX )
1

Intermediate pressure, PHX at the intercooler;

PHX = P1 P4 = (100 kPa)(400 kPa)

PHX = 200 kPa


Solving for the volume displacement of the first stage, VD1;
1
V1 200 kPa 1.4
= VD1
= 1 + 0.08 0.08 (100 kPa) = 0.9487 = 94.87%
V
= V 1
D1

m3
0.05
s
0.9487 = VD1

m3
VD1 = 0,05270 s

Substituting;

VD1 = ( 4 d1 2 ) LN

m3 m
0.05270 s
= ( 4 d1 2 ) (1.25 s )
100 cm
d1 = (0.231 m) ( 1m
)

= .
Solving for the diameter of the piston at the second stage, d2;

VD2 = ALN = ( d2 2 ) LN
4
Solving for the volume displacement of the second stage, VD2; and knowing that the volumetric
efficiency is equal at the first stage and the second stage of compressor;
V
= V 3
D2

Solving for V3;


At the suction of the first stage: P1 V1 ; = mRT1
At the suction of the second stage: P3 V3 ; = mRT3
Therefore;
P3 V3 ; = P1 V1 ;
m3
(200 kPa)V3 ; = (100 kPa) (0.05 )
s

m3
V3 ; = 0.025
s

Substituting to solve for the volume displacement of the second stage;


V
= V 3
D2

m3
0.025
s
0.9487 = VD2

m3
VD2 = 0.02635 s

Substituting;
m3 m
0.02635 s
= ( 4 d2 2 ) (1.25 s )
100 cm
d2 = (0.1638 m) ( 1m
)

= .

In summary:
= .
= .
b) Solving for the total power required, WC12
k1
2kP1 V1 PHX k
WC12 = [( ) 1]
k1 P1

m3 1.41
2(1.4)(100 kPa) (0.05 ) 200 kPa 1.4
s
WC12 = [( ) 1]
1.4 1 100 kPa

= .

c) Solving for the heat loss in the intercooler, QHX;


From the intercooler:
Q HX = ma CP (T1 THX )
For THX;
k1
THX P k
T1
= ( PHX )
1

1.41
THX 200 kPa 1.4
T1
= (100 kPa)

THX = 1.2190T1
For ma;
P1 V1 ; = ma RT1
m3 kJ
(100 kPa) (0.05 ) = ma (0.287 kgK) (T1 )
s

17.4216
ma =
T1

Substituting;
17.4216 kJ
Q HX = ( T1
) (1.0062 kgK) (T1 1.2190T1 )


= .

Example:
A reciprocating compressor receives air at 100 kPa and 0.20 m3 of air per second and delivered at 1000
kPa. Assumimg conditions are ideal, and the compression and expansion has a polytropic exponent of 1.3,
determine the savings in compressor work kW, due to (a) two staging and (b) three staging.
1. Solving for the work required by the compressor for single stage, WC1;
n1
nP1 V1 P2 n
WC1 = [( ) 1]
n 1 P1

m3 1.31
(1.3)(100 kPa)(0.20 ) 1000 kPa 1.3
s
WC1 = 1.31
[( 100 kPa ) 1]

= .
Solving for the work required by the compressor for two stage, WC12 ;
n1
2nP1 V1 PHX n
WC12 = [( ) 1]
n1 P1

PHX = P1 P4 = (100 kPa)(1000 kPa) = 316.2278 kPa


m3 1.31
2(1.3)(100 kPa)(0.20 ) 316.2278 kPa 1.3
s
WC12 = 1.31
[( 100 kPa ) 1]

WC12 = 52.7490 kW
Solving for the compressor work savings, in kW:
Work savings = (60.7754 52.7490) kW
Work savings = 8.0264 kW

2. Solving for the work required by the compressor for three stage,
WC123 ;
n1
3nP1 V1 PHX n
WC123 = [( ) 1]
n1 P1

3 3
PHX = P1 2 P6 = (100 kPa)2 (1000 kPa) = 215.4435 kPa

m3 1.31
3(1.3)(100 kPa)(0.20 ) 215.4435 kPa 1.3
s
WC123 = 1.31
[( 100 kPa ) 1]

WC123 = 50.3818 kW

Solving for the compressor work savings, in kW:


Work savings = (60.7754 50.3818) kW
Work savings = 10.3936 kW
We can say that by having multiple compressor stages decreases the required work of
the compressor, thus, increasing the power savings, in kW.
General Formula for Multi-stages Compressor,(m number of stages)
The compressor work, WC m is given by;
n1
mnP1 V1 PHX n
WC m = n1
[( P ) 1]
1

The intermediate pressure, PHX can be theoretically approximated by:


m
PHX = (P1 )m1 PF
Where: P1 = suction pressure
PF= final or discharge pressure

COMPRESSOR SELECTION
The following are important items to consider in selecting a compressor:
The desired capacity, V1
The required discharge pressure
The foundation required
The characteristics of gas to be handled
Type of control required
Other things to be considered in the selection are the space requirements, availability of
intercooler cooling water, maintenance costs, power source and economics.
Reciprocating compressors are generally driven by a belt. Similar with belt driven fans, one
advantage is that the belts tend to slip that reduce load in the motor during start up. But on the other
hand, maintenance cost could be high in belt driven compressors because belts are designed to break
to some degree of usage.

COMPRESSOR INSTALLATION
The following are some of the guidelines and precautions in installing a compressor. It is also
important to consult the compressor manufacturer for additional recommendation and further
information regarding the compressor to be used. These outlines are similar with the other fluid
machineries installation described in previous chapters.
It is necessary to have a good foundation for the compressor unit in order to stabilize
the vibration that occurs during its operation.
Inlet filter should be installed in the suction of the compressor to protect the unit from
dust, foreign object, moisture and corrosive elements present that may damage the
compressor.
To reduce friction and turbulence, all compressed air piping system lines should be
short and straight as possible, with minimum elbows, valves and fittings.
One of the challenges in maintaining compressed air piping system is the collection of
water in the pipelines. A suitable water drain or collector should be available at each
point in the line
Install the compressor in a secure location and at the same time, available for
inspection and maintenance.
Silencers may be used if noise level is a primary consideration

Air Receivers
It is necessary for all reciprocating compressor to have an air receiver to eliminate the
pulsation of air delivered. It is also used to store compressed air, and to condense some of its
moisture content by cooling it down.
Air receiver size, VT can be theoretically calculated from;
V1 P1
VT = P2

Air receiver size depends on the capacity of the compressor, pipeline network and
pressure drop, air consumption of the plant and switching cycle per unit-time of the
compressor operation.

PROBLEM SET:
1. An air compressor delivers air at a flow rate of 0.25 m3/s with initial pressure of 100 and
discharge pressure of 680 kPa. Determine the power of the compressor. (Ans. 63.8)

2. A double acting compressor with 225 mm x 380 mm cylinder runs at 480 rpm with a clearance
of 8%, compressor air 7 times its initial pressure. Compute the compressor capacity in cubic
feet per minute. (Ans. 388.3 ft2/min)
3. A single stage compressor with initial pressure of 100 kPa and discharge pressure of 850 kPa
has a suction volume of 0.3 m3/s. Determine the percent decrease in power due to two staging
if the compressor process is PV1.33 = C. (Ans. 13.2%)

4. An air compressor takes air at 98 kPa at a rate of 0.4 m3/s and delivers it at a pressure of 620
kPa. If the power input to the compressor is 135 kW, find the heat loss in the compressor. (Ans.
39.8 kW)

5. A single stage compressor with a suction pressure of 15 psi discharge ar at a pressure of 75 psi.
If the suction volume is 110 ft3/min, determine the horsepower capacity of the motor needed
to drive the compressor if the compressor efficiency is 80%. (Ans. 183.9 hp)

6. A two stage compressor has a suction volume of 720 m3/hr at 100 kPa and 26C, the discharge
is 700 kPa, determine the heat rejected at the intercooler. (Ans. 22.5 kW)

7. An air compressor has a suction volume of 720 m3/s and discharges to 650 kPa. If the power
input to the compressor is 120 kW, find the heat loss in the compressor. (Ans. 5.3 kW)

8. A two stage compressor takes air at 98 kPa and 24C with a volume flow rate of 0.122 m3/s and
discharges to 680 kPa. What is the amount of heat rejected at the intercooler? (Ans. 13.4 kW)

9. The piston speed of an air compressor is 155 m/min and the displacement volume is 0.25 m3/s.
Find the diameter of the compressor cylinder. (Ans. 504.6 mm)

10. Determine the volume displacement of a double acting compressor that has cylinder dimension
of 52 cm x 65 cm and runs at 660 rpm. (Ans. 3.04 m3/s)

11. An air compressor has suction condition of 98 kPa, 28C and 0.22 m3/s. If the surrounding air
100 kPa and 23C, calculate the free air capacity in m3/s. (Ans. 0.212 m3/s)

12. An air compressor has a suction volume of 0.35 m3/s at 28C and 101.325 kPa and discharges
to 680 kPa. Determine the amount of power saved by the compressor in two staging? (Ans.
14.3 kW)

13. The initial condition of air is 100 kPa and 24C and compressed to 550 kPa. The bore and stroke
are 365 mm and 380 mm, respectively. If the percent clearance is 8% and runs at 320 rpm,
determine the mass flow rate of air at the suction, (Ans. 0.20 kg/s)

14. A 15 hp motor is used to drive an air compressor. The compressor efficiency is 75% and the air
is available at 29%. Find the mass of air needed if the pressure compresses to 7 times initial
pressure. (Ans. 2.23 kg/min)

15. The discharge pressure of an air compressor is 6 times the suction pressure. If the volume flow
at the suction is 0.6 m3/s, find the power required by the compressor operating at PV1.33 = C
with a suction pressure of 100 kPa. (Ans. 135.4 kW)
CHAPTER 2
Fans and Blowers

Fans and blowers are machines that move air or gases under moderate pressures. They are significant
and important component in all industrial plant specifically if there is a need in supplying, circulating and
removing air in a space in order to provide comfort and safety. They are also utilized for heating, ventilating, air
conditioning and pollution control needs for industrial plants.

FIGURE 2.1
Industrial Fans (Picture
courtesy of Pollrich DLK
Fan Factories)

Although fans
and blowers,
including compressors (to be
discussed in the next chapter) performs a similar function of adding energy
and moving gasses, they all differ in terms of their pressure ranges.
Generally, fans works at low pressures from 1 psig, while blowers (also known as turbo-blowers) works
up to 35 psig and compressors starts at 35 psig.

TYPE OF FANS
Two general types of fans are Centrifugal Fans and Axial Fans
1. Centrifugal Fans
Centrifugal fans moves air radially thru the impellers. Air drawn at the center of a
revolving wheel, connected to a rotating shaft, and then enters the spaces of the wheel blades.
The air is then thrown out perpendicularly to the impellers axis of rotation at high velocity
and static pressure. They are widely used in comfort applications because its quite, efficient
and operates at relative high pressure heads.
Centrifugal are classified in terms of wheel blades: (1) Forward- curved, (2)
backward curved and (3) radial

FIGURE 2.2 Centrifugal Fan

2. Axial Fans
Axial fans moves air in a helical-type flow pattern that is parallel thru the
impeller. This type of fan moves large amount of fluid and develops small pressure
heads. They are widely used in applications where noise levels are of secondary.
Axial fans are classified as: (1) Propeller, (2) tubeaxial and (3) vaneaxial

TYPES OF CENTRIFUGAL FANS


Forward-Curved Fan
A forward-curved fan or also known as squirrel cage fan is generally used for high-volume air flow
applications and low to medium static pressures applications. Widely used in heating and ventilating work
because of its quite operation, fan should be operated at relatively clean air because contaminants might clog
its blades.

A typical performance curve of forward-curved fan is shown in Figure 2.3. As we can observe, the
horsepower is at minimum value at no delivery and the highest efficiency occurs when the fan is operating at
35-45% of free delivery capacity. Furthermore, the behaviour of the horsepower input curve increases as the air
delivered increases, while the static pressure decreases. Forward-curved fan is overloading fan because the
motor may overload when the static pressure is below its design value.

FIGURE 2.3: Forward-Curved Blade Performance Curve


and Blade Profile

An example of this is, lets say, a fan is designed to operate at 40% of free delivery capacity and at 95%
static pressure. The motor installed for this fan to operate properly is say, with maximum capacity of 4 bhp and
is operating at 3.5 bhp.
If fir instance, the fan access door is removed, the static pressure or the system resistance will drop, say
at 30% and at the same time the free air delivered will increase to 80%, at this point the motor must operate at
a higher rate, say a 5 bhp, which is greater than the power capacity of the installed motor, in this case the motor
will stop on overload.

Backward-Curved Fan
Backward-curved fan or also known as limit loading fan because of the backward profile of its blade
and is widely used for high-static pressure applications. This type of fan is more efficient than forward-curved
with efficiency between 50-70% of free delivery capacity, and is quieter than any other types of fan.
A typical performance curve of a backward-curve fan is shown in Figure 2.4. It has a minimum
horsepower at no delivery. The horsepower curve gradually increases at the range of maximum efficiency and
then gradually decreases.
Backward-inclined and airfoil blade fan are modifications of backward-curve fan. Airfoil blade fan has a
high efficiency because of its aerodynamic blade profile which allows smooth flow of air thru its blades.

FIGURE 2.4:
Backward-Curved Blade
Performance Curve and
Blade Profiles and
Modifications

Radial Fan
Radial
(straight) fans are
widely used in waste
collection systems
that require high velocities and high pressure heads. This type of fan is preferred for
high dust loading in gasses because of its self-cleaning wheel design and well
suited for high temperature applications.
A typical performance curve of a backward-curve fan is shown in Figure 2.5. The horsepower input curve
increases as the air delivered increases, while the static pressure decreases. This fan is an overloading fan and
the least efficient of all centrifugal fans.

FIGURE 2.5: Radial Blade Performance Curve and Blade Profile

TYPES OF AXIAL FANS


Propeller Fan
Propeller fans consist of two or more blades and are widely used for general ventilation systems in which
require large volume of air at low pressure heads. It is used ad exhaust system for indoor applications and for
air-cooled condensers and cooling towers for outdoor applications.
Propeller fans are relatively noisy because of the turbulence created when air is discharged in a circular
or helical pattern. This type of fan has a simple ring enclosure for its housing and no ducting system is required.
A propeller fan operates generally at 0 to 1 inch of water gage or less.

FIGURE 2.6: Propeller Fan

A typical performance curve of a backward-curve fan is shown in Figure 2.7. The horsepower decreases
as the percent of free delivery increases and the lowest at maximum percent of free delivery. Horsepower also
decreases as the static pressure decreases. The highest efficiency of propeller fans occur when it is delivering
50%-70% free delivery capacity.

FIGURE 2.7: Propeller Performance Curve


Tubeaxial Fan
Tubeaxial fans are axial fans that are fabricated in a tubular casing. They are widely used in ducted
ventilating applications that require medium pressure heads and where air flow pattern in downstream of the
fan are not of primary consideration, such as in fume exhaust system, drying ovens and paint spray bottles.
The blades of tubeaxial fans are similar to propeller fans, except that it usually has 4 to 8 blades and is
designed for heavy duty applications.

FIGURE 2.8 Tube Axial Fan

Typical performance curve of axial fans are presented in Figure 2.9. The efficiency and static pressure
curve patterns of tubeaxial fans has a significant improvement compared with propeller fans.

FIGURE 2.9: Axial Flow Performance Curve

Vaneaxial Fan
Vaneaxial fans are tubeaxial fans with guide vanes. They operate in medium to high pressure heads and
are widely used when good stream air distribution is needed.

FIGURE 2.10 Vaneaxial Fan

The general advantages of axial fans are in terms of its simplicity and its installation, small space
requirements and economy. They are widely used in applications where high volume of air is needed against a
low pressure heads and noise is not a primary consideration.

FAN FUNDAMENTALS
Air Power, AP
Air power is the energy added by the impeller of the fan to air or gasses to move it against a particular
pressure.
Air power is referred to as total air power when total pressure head, HT is used, and as static air
power, when static pressure head, hs is used in the equation.

Total Air Power


AP = Qa HT

g ma go
a = a [go ] = [ ]
va gc
c

g
AP = ma [go ] HT
c

Where: AP = Total air power, kW or hp


1 hp = 0.746 kW
Q = v = volume flow rate, m3/s or ft3/sec
a = specific weight of air
For air at standard condition @ 21C and 101.325 kPa:
SI units: 0.0118 kN/m3
Eng units: 0.075 lbf/ft3
kg
ma = mass flow rate of air, s
or lbm/sec

HT = total pressure head of the


fan meters of air or feet of air

Static Air Power

APstatic = Qa hs

g
APstatic = ma [ga ] hs
c

Where: APstatic = Static air pressure, kW or hp


hs = static pressure head of the fan meters of air or feet of air

The Total Fan Pressure Head, HT


Total fan head, the total amount of work needed by the fan (usually measured in meter or feet) per
specific weight of air flowing through the fan.
It is the sum of the velocity head and static pressure head.
hw(Water Gage, WG)

Manometer Fluid: Water SG = 1

FIGURE 2.11 Fan configuration showing


velocity and static pressure head
AIR FLOW
AIR FLOW

Consider a fan configuration in Figure, applying conservation of energy:


[Ein = Eout]
PE1 + KE1 + Wf1 + U1 + AP = PE2 + KE2 + Wf2 + U2
AP = (PE2 PE1) + (KE2 KE1) + (Wf2 Wf1) + (U2 U1)

If we assume that the temperature and elevation from the suction and
discharge of the pump are almost equal, t1 t2 and z1 z2, then we can say
that the change in internal energy and potential energy are negligible, U 0
and PE 0

1ma
AP = (v2 2 v1 2 ) + Va (P2 P1 )
2gc

g
Multiplying both sides of the equations by m cg
a

gc 1(v2 2 v1 2 ) (P2 P1 )
ma g
AP = 2(go )
+ ma go
[ ]
va gc

Therefore:
1 (v2 2 v1 2 ) (P P )
HT = + 2 1
2 go a

HT = (velocity head) + (static pressure head)


Or;
HT = hv + hs

Velocity Head, hv
Velocity head is the energy possessed by air because of its velocity. This has a significant effect
in fan performance and must be considered in the calculation.
Velocity head is given by the equation:
1 (v2 2 v1 2 )
hv = 2 go

Static Pressure Head, hs


Static pressure head is the energy required to overcome resistance. Static pressure head
becomes irrelevant when the fan operates against no resistance (e.g. no ducting systems or dampers) and when
the fan outlet velocity is high.
To determine the capacity of the fan to be installed, it is necessary to know the resistance
characteristics of the system. A fan operating in a system with narrow ducting systems, multiple short elbows,
bends and twists will require more power to overcome system resistance, compared with a fan operating with
a larger ducting system and minimum elbows, bends and twists.
Static pressure head is given by the equation:
(P2 P1 ) w hw
hs = a
= a

Therefore, we can say that static pressure head is also equivalent to:
w
hs = hw a
w
Or; hs = hw a
Where:
w = density of water
For water at standard condition
SI units: 1000 kg/m3
Eng units: 62.4 lbm/ft3
a = density of air
For air standard condition @ 21C and 101.325 kPa
SI units: 1.2 kg/m3
Eng units: 0.075 lbm/ft3

Example:

Air enters a fan at an initial velocity of 7 m/s and a static vacuum pressure head of 22.5 mm of
water. The air is delivered at a velocity of 12 m/s and a static pressure head of 81 mm of water. The fan
has a capacity of 9.5 m3/s and is coupled to a 15 kW motor. Determine the total air power if the density
of air is 1.2 kg/m3.
Solving for the total air power, AP;
AP = Qa HT
Solving for the total pressure head;
Ht = hv - hs
2 2 m2
1 (v2 2 v1 2 ) 1 (12 7 ) s2
hv= = = 4.841997961 m of air
2 go 2 9.81 hs
m

(P2 P1 )
hs = a

kN
P2 = w hw2 = (9.81 ) (0.081m) = 0.74461 kPa
m3
kN
P1 = w hw1 = (9.81 m3
) (0.0225m) = 0.220725 kPa
m
g kg 9.81 2 kN
a = a [ga ] = (1.02 m3 ) ( s
kgm ) = 0.011772 m3
c 1000
kNs2

(0.79461+0.220725)kPa
hs = kN = 86.25 m of air
0.011772 3
m

HT = (4.8420 + 86.0424) m of air


HT = 90.8844 m of air
Substituting HT, to solve for AP;
m3 kN
AP = Qa HT = (9.5 s
) (0.011772 m3 ) (91.091999 m)
= .

Example:

Air is flowing with an initial and final velocity of 1.2 m/s and 8.3 m/s, respectively, against a static
pressure of 2.25 cm water gage. The duct diameter is 1.5 cm and the condition of air is at 99.5 kPa and
30C. Determine the total pressure in which the fan must operate (a) in meters of air and (b) in
centimeters of water.

(a) Solving for the total pressure in meters of air

HT = hv + hs
2 2 ms
1 (v2 2 v1 2 ) 1 (8.3 1.2 ) s2
hv = 2
[ go
] 2
m = 3.4378 m of air
9.81 2
s

w
hs = hw
a

m P 99.5 kPa kg
a = v
= RT = KJ = 1.1442 m3
(0.287 )(30+273)K
kgK
kg
(1000 3)
m
hs = (0.0225m) kg = 19.6644 m of air
(1.1442 3)
m

HT = (3.4378 + 19.6644) m of air


= .
(b) Solving for the total pressure in centimeters of water

HT = hv + hs
kg
(1.1442 3)
hv = (3.4378 m of air) m
kg = 3.9335 103 m of water
(1000 3)
m

hv = 0.39335 cm of water
Hs = 2.25 cm of water

= .

Fan Efficiency
The fan efficiency is the ratio of the air power output of the fan over the brake power input to
the fan. It shows how effective the fan in converting brake power input into air power. Consider the fan shown
in Figure 2.12:

FIGURE 2.12 The as the system

There are two ways on how manufacturers describes the fan efficiency: the (1.) fan total or
the mechanical efficiency and (2.) fan static efficiency.
The fan efficiency is known as Fan total efficiency, if total air power is used, and as Fan
static efficiency, if static air power I used.
Fan total efficiency:
AP Qa HT
F = BP 100% = BP
100%

Fan static efficiency:


APstatic Qa hs
Fstatic = BP
100% = BP
100%

Example:
Determine the brake horse power required for a fan with a static efficiency of 50% and delivers
air at 1200 cfm. The static pressure developed is 2.5 inches of water. Air is at 60F and barometric
pressure of 30 inches mercury.

Solving for BHP;


APstatic
Fstatic = BHP
100%

APstatic = Qa hs
w
Specific weight of air, hs = hw a

1200 ft3 lb 2.5


APstatic = Qw hw = ( 60 sec
) (62.4 ft3f) ( 12 ft)

lbf ft 1hp
APstatic = (260 sec
) ( lbfft) = 0.4727 hp
550
sec
Therefore;
0.4727 hp
0.50 = BHP

BHP = 0.9455 hp

Example:
A fan with a capacity of 1.5 m3/s draws air at a static pressure of 3.0 cm of water through a duct.
The air drawn is measured at 27C and 760 mmHg. If the inlet and outlet duct is 320 mm and 280 mm,
respectively, determine the fan static efficiency when the total fan efficiency is 80%.
Solving for fan static efficiency,
APstatic
Fstatic = BP
100%

APstatic = Qa hs and hs = hw w , hence;
a

m3 kN
APstatic = Qw hw = (1.5 s
) (9.81 m3 ) (0.03 m)

APstatic = 0.4415 kW

Solving for the brake power input, BP, from total fan efficiency;
AP
F = BP 100%

AP = Qa HT
HT = hv + hs
1 v2 2 v1 2
hv = 2 ( go
)

Solving for the velocity of air at the inlet and exit ducts, using;
Q = A1 v1
m3 (0.320 m)2
1.5 s
= 4
v1

v1 = 18.6510 m/s
Q = A2 v2
m3 (0.280 m)2
1.5 s
= 4
v2

v2 = 24.3605 m/s

Solving velocity head, hv


2 2 m2
1 (24.3605 18.6510 ) s2
hv = 2 m 2 = 12.5165 m of air
9.81
s2

Solving for the static pressure head, hs


w
hs = hw a

m P 101.325 kPa kg
a = v
= RT = kJ = 1.1768 m3
(0.287 )(27+273)K
kgK

m
g kg 9.81 2 kN
a = a [go ] = 1.1768 m3 [ s
kgm ] = 0.01154 m3
c 1000
kNs2

kN
9.81 3
m
hs = (0.03 m) ( kN ) = 25.5026 m of air
0.01154 3
m

The total fan pressure head; HT


HT = (12.5165 + 25.5026) m of air
HT = 38.0191 m of air
For total air power, AP;
m3 kN
AP = (1.5 ) (0.01154 3 ) (38.0191 m)
s m

AP = 0.6581 kW
Substituting to the total fan efficiency, to solve for the brake power input, BP;
0.6581 kW
0.08 = BP
100%

BP = 0.8226 kW
Then for the fan static efficiency;
0.4415 kW
Fstatic = 100%
0.8226 kW

= . %

FAN SPECIFIC SPEED


Fan specific speed is a dimensionless parameter used to describe the applications of different
types of fans. It is defined as the revolution per minute at which a given geometrically similar impeller
of a fan would operate if reduced proportionally in size so as to deliver a rated capacity of 1 cubic feet
per minute against a differential head of 1-foot of air under standard conditions.
NQ
Ns = 3
hs 4

Where: Ns = specific speed of the fan, rpm


Q = capacity of the fan, cfm
hs = fan static pressure head, ft. of air
N = impeller speed, in revolution per minute
FAN TYPE AND SPECIFIC SPEED
Figure shows the specific speed of centrifugal and axial fans based in speed, fan capacity and static
pressure. It shows that forward-curved fans generally operate at low speeds while propeller fans operate at high
speeds to attain its peak efficiencies.

FIGURE 2.13: Specific speed ranges (ref. Handbook of Air conditioning System Design Carrier Air Conditioning Design Company)

FAN AFFINITY LAWS


Fan affinity laws are rules that express the relationship of fan capacity, head (either static or total),
BHP input and air density when one of each parameter is changed. In applying the following equations, we
consider that the efficiency is the same for both conditions.
Variation in speed Variation in density Variation in density
N1 N2;1 = 2 N1 = N2;1 2 N1 N2;1 2

Capacity Q1 N1 Q1 = Q 2 Q1 N1
= =
Q 2 N2 Q 2 N2
Head h1 N1 2 h1 1 h1 1 N1 2
=( ) = = ( )
h2 N2 h 2 2 h 2 2 N 2

BHP BHP1 N1 3 BHP1 1 BHP1 1 N1 3


=( ) = = ( )
BHP2 N2 BHP2 2 BHP2 2 N2

Where: Q = Capacity, in gpm


h = Total head (static or total), in feet
BHP = Brake horsepower, in hp
N = Fan speed, rpm
Subscript 1 refers to initial condition and subscript 2
refers to the new condition.
Example:
Determine the performance of a fan with a static pressure head of 6.20 cm of water, speed of 410 rpm
and initial power input of 3 kW. The capacity is changed from 5.5 m3/sec to 8.6 m3/sec.
In this example problem, fan capacity (which is a function of motor speed) is changed while maintaining
a constant air density;
Q1 N
Q2
= N1
2

m3
5.5 410 rpm
s
m3
= N2
8.6
s

N2 = 641.0909 rpm
h1 N 2
= ( 1)
h2 N2

6.2 cm of water 410 rpm 2


H2
= (614.0909 rpm)

h2 = 15.1587 cm of water
BP1 N 3
= ( 1)
BP2 N2

3 kW 410 rpm 3
BP2
= (614.0909 rpm)

BP2 = 11.4691 kW
Example:
A fan delivering air through a ducting system that has a capacity of 100m3/s, static pressure head of 30
cm of water, speed of 420 rpm and initial power output of 600 kW. Determine the performance of the fan if the
air is to be delivered at 85C and a barometric pressure 734 mmHg.
In this example problem, the fan speed and capacity remains constant while the air density is changed because
of the increase in temperature from standard air temperature of 21C to 85C.
Q1 = Q 2
Q1 = 100 m3/s
h1 1
=
h2 2

kg
1 = 1.2 m3 at standard condition of 101.325 kPa and 21C.
101.325 kPa
m P2 (734 mmHg)( )
760 mmHg
2 = = = KJ
v RT2 (0.287 )(85+273)K
KgK

kg
2 = 0.9524 m3
kg
30 cm of water 1.2 3
m
h2
= kg
0.9524 3
m
h2 = 23.81 cm of water
BP1
BP2
= 1
2

kg
600 kW 1.2 3
m
BP2
= kg
0.9524 3
m

BP2 = 476.20 kW
Example:
A fan has a capacity of 80 m3/s, static pressure head of 22 cm of water, speed of 380 rpm and initial
power input of 400 kW. Determine the performance of the fan when the air is to be delivered at 70C and at the
speed of 410 rpm.
In this example problem, the fan speed and air density is changed.
Q1 N
Q2
= N1
2

m3
80 380 rpm
s
Q2
= 410 rpm

Q2 = 86.3158 m3/s
h1 N 2
h2
= (N1 )
2

kg
1 = 1.2 m3 at standard condition of 101.325 kPa and 21C.
m P 101.325 kPa
2 = v
= RT2 = KJ
2 (0.287 )(70+273)K
KgK

kg
2 = 1.0293 m3
kg
22 cm of water 1.2 3
m 380 rpm 2
h2
= kg (410 rpm)
1.0293 3
m

h2 = 21.9677 cm of water
BP1 N 3
BP2
= 1 (N1 )
2 2

kg
400 1.2 3
m 380 rpm 3
BP2
=( kg ) (410 rpm)
1.0293 3
m

BP2 = 430.9447 kW

FAN SELECTION
The following are important items to consider in selecting a fan:
The desired flow rate, Q
The static pressure head, hs
The characteristic of air to be handled (e.g. dust concentration)
Operating temperature
The prevailing noise level of the space to be served
Other things to be considered in the selection are the space requirements, maintenance costs, space
requirements, power source and economics.
Fans can be directly coupled to a motor or driven by a belt. One advantage of belt driven fans is that the
belts tend to slip that reduce load in the motor during start up. Maintenance cost could be high in belt driven
fans because belts are designed to break to some degree of usage.
Sometimes it is necessary to vary the flow rate of air to balance the flow rate of air delivered in the
system. This can be accomplished by varying the speed of the motor. Or, if the fan is belt driven, a change in
the drive pulley can also be made. Another method is by installing dampers in the duct system, which increases
the static pressure of the fan.

FAN INSTALLATION
The following are some of the guidelines and precautions in installing a fan. It is also important to consult
the fan manufacturer for additional recommendations and further information regarding the fan to be used.
These outline are similar with the outline given for pump installation. The only difference is the type of fluid they
handle and the Net Positive Suction Head, NPSH considerations are critical for pump installation.
To reduce friction and turbulence, all ducting lines should be short and straight as possible, with
minimum elbows, valves and fittings.
Elbows in ductings should have small radius.
Both the suction and discharge lines should be supported independently to protect the casing
from strains that may cause distortion to the fan. Flexible ducting system can also be used to
isolate the vibration of the fan.
Install the fan away from heat source if the fan takes air from the atmosphere so that the air will
be cooler.
The suction of the fan should be protected from dust, foreign object, moisture and corrosive
elements present that may damage the fan.
Silencers may be used if the noise level is primary consideration.
The inlet damper positioning should be checked regularly to satisfy the closed and fully open
condition of the fan. Install the fan in a secure inspection and maintenance.

FANS IN SERIES AND PARALLEL


Similar with pumps, fans can also be installed in series and or in parallel operation.
Fans in Series
Fans in series are done by staging multiple fans as shown in Figure 2.14. The total pressure is
increased at a given capacity as shown in the performance curve.

TWO FANS IN SERIES

FIGURE 2.14: fans installed in series

Fans in Parallel
Fans in parallel are the result of installing multiple fan side by side as shown in Figure 2.15. The
capacity is doubled while maintaining the total pressure.
TWO FANS IN PARALLEL

FIGURE 2.15: Fans installed in parallel

PROBLEM SET:
1. The power output of a fan is 120 kW with efficiency of 80%. Determine the horsepower output
required by the motor to drive the fan. (Ans. 201 hp)

2. The static head of a fan is measured to be 160 mm of water gage at an air velocity of 25 m/s. Find
the air power at an air condition of 28C and 98 kPa with a volume flow rate of 5 m3/s. (Ans. 9.6 kW)

3. Determine the horsepower required for a fan delivering 35 ft/sec of air through a 2.3 ft x 3.5 ft duct
with a total pressure of 3.5 in water gage. Take density of air to be 0.075 lb/ft3. (Ans. 9.3 hp)

4. A fan operating at a standard air condition registered a total static head of 230 mm of water gage.
If the static efficiency is 65% and the fan efficiency is 80% determined the velocity of air if the volume
of air delivered is 6 m3/s. (Let the velocity head to be 35% of the static head). (Ans. 34.6 m/s)

5. A fan initially operating at a speed of 380 rpm at an air temperature of 26C. If the speed is increased
to 460 rpm with 55C, determine the new head in mm of water gage for an initial head 190 mm of
water gauge. (Ans. 253.8 mm)

6. What is the horsepower required for a fan to deliver 230 ft3/sec of air through a 2.5 ft x 4.5 ft duct
under a total pressure of 4.2-in water gage? (Ans. 0.35 hp)

7. The rating of a fan is 610 m3/min when running at 360 rpm and requires 8 kW motor to drive it. If
the fan speed is increased to 620 rpm and the air handled becomes 60C instead of 30C, determine
the power in kW. (Ans. 37.1 kW)

8. A fan has a static head of 110 m at a pressure of 1.5 kg/cm 2 and 75F. If the air velocity is 18 m/s,
determine the total equivalent head in mm of water gage. (Ans. 145.61 mmWG)
HYDRAULIC TURBINES
Fluid machines are those machines which are used to convert mechanical energy into fluid energy
or vice versa. Machines that coverts fluid energy into mechanical energy are known as turbines.
Machines that converts mechanical energy into fluid energy are known as pumps.
Water turbines. Turbines are devices to convert energy of water into mechanical energy which can
be used for running electricity generators.

Types of Hydraulic Turbines


1) Reaction turbines
2) Impulse turbines

In reaction turbines, pressure head of water is converted into velocity head as water flows through
the turbine. Reaction turbines run full of water and hence the turbine maybe entirely submerged below the tail
race or may discharged into the atmosphere. It may also discharge into a suction or draught tube when placed
9.14 m above the foot of fall. In these turbines water must enter over the whole circumference.

In impulse turbine, the energy of water is converted into velocity before entering the wheel at
atmospheric pressure and this will not permit the turbine tube to be flooded. The water is passed through nozzle
or guide vanes for converting all its energy into velocity. Impulse turbine must be placed at the foot and above
the tail race. Hence these turbines can be inspected easily. The water maybe entered over entire circumference
or part of the circumference.

If H is the head at which water is available and leaves the turbine at velocity V1, then the energy
available from turbine or energy of water absorbed by turbine will be;

12
H=
2

However, the impulse turbine, this energy H will first be converted into velocity or kinetic energy at velocity V.

Both reaction and impulse turbines can be further classified depending upon direction of flow. The flow is axial
in axial flow turbines and radial and radial flow turbines. Mixed flow turbines are also in used. The radial flow
turbine may have flow from centre to circumference or vice-versa.

Commonly used turbines are:

a) Kaplan turbine/Propeller type, reaction turbine suitable for very low heads.
b) Francis turbine. A reaction turbine suitable for medium heads.
c) Pelton wheel. An axial flow impulse turbine for high heads
d) Turgo wheel. An impulse turbine suitable for medium heads.

In an inward radial flow turbine fixed guide blades surround the revolving blades externally whereas in an
outward flow turbine the ring of moving blades surrounds the fixed guide blades The water leaves through the
centre in latter case while it enters the turbine though centre in former. In outward flow turbine the relative
velocity of water is increased and hence quantity of water passing through the turbine is increased. This
tendency would not exists in inward flow turbines. In fact, this case may increase the speed of the wheel will
tend to decrease the flow through the wheel and reduce power.

In axial flow turbine, the guide blade and rotating blade rings are mounted side by side and water flows from
guide blades into moving blades parallel to the axis of turbine. For this reason the turbines are called parallel
flow.

Types of Hydraulic turbines

1.0)Impulse turbine also known as Tangential wheel or Pelton wheel It is a Pressureless Turbine

__ a turbine that utilizes kinetic energy of high velocity jet which acts upon a small part of the circumference at
an instant.

__ Movement of the water is tangential


__ Suited for very high heads plants (150 m and above) and low volume of water

__ No exact value for critical head, hence heads are given in range

__Impulse turbine has no draft tube

__ Typical turbine efficiencies are in the range of 80% to 90%

__Higher efficiencies are associated with turbines having two or more runners.

2.0) Reaction Turbines (Francis type or Propeller type). It is Pressure Turbine

__ a turbine which develops power from the combined action of pressure and velocity of water that completely
fills the runner and the water passages

__Movement of water for reaction turbines can be radial for Francis type and axial for Propeller/Kaplan type

__Reaction turbine has draft tube which keeps the turbine up to 5m( 15 ft) above the tail water

__Reaction turbine conversion efficiency is usually higher than that of Impulse turbine

__Reaction turbines conversion efficiency is about 85% to 95%

Turbine type recommended based on head

Net head Type of turbine

Up to 70 ft Propeller type

70 to 110 ft Propeller or Francis type

110 to 800ft Francis type

800 to 1300 ft Francis type or Impulse type

1300 and above Impulse type

Types of reaction turbines

A. Francis Type--- for medium head


B. Propeller and Kaplan type Reaction turbine for very low head
Propeller has fixed blade. A type of reaction turbine with reduced number of fixed blades. The flow is
axial. Suited for low head plant and has usual conversion efficiency of 80%
Kaplan type has adjustable blades. The flow is inward flow axial. Suited for low headed and large volum
e of water and usual conversion efficiency of more or less than 93%.

Performance of Hydroelectric Power Plant


1. Gross head, hg---- is the difference between water and tail water elevations .
hg= hhw - htw
where: hhw =headwater elevation
htw = tail water elevation
2. Friction head loss,hf
Friction head loss- is the head lost by the flow in a stream or conduit due to frictional disturbances
set up by the moving fluid and its containing conduit and by intermolecular friction.
Using Darcy equation
2
hf= 2
Using Morse equation
2 2
hf =
where: hf= friction head in meter
f = coefficient of friction
L = total length in m
g = 9.81 m/s2
D= inside diameter in m
Note: Friction head loss is usually expressed as a percentage of the gross head
3. Net head or Effective head is the difference between the gross head and the friction head loss
H = hg-hf
4. Penstock efficiency is the ratio of the net head to the gross head

ep =
5. Volume flow rate of water, Q it is the product of the velocity and the cross-sectional area.
Q=AV
6. Water Power- is the power generated from an elevated water supply by the use of hydraulic turbines
7. Turbine efficiencyis the ratio of turbine power output to the water power output

et = = , and Pt = Qhet
8. Electrical or generator efficiency is the ratio of the generator output to the turbine power output

eG = = ; Pgen =Ptegen = Qhetegen

120
9. Generator Speed, N = ,
where: N= angular frequency, rpm
F= frequency (usually 60 hertz), P = no. of poles,(even number)
10. Head of impulse turbine
2
h = + 2 meters of water
11. Head of reaction turbine
2 2
H= + Z + 2
12. Peripheral coefficient
It is a ratio of the peripheral velocity(Vp) to the velocity of the jet (Vj)

= where: D= diameter of the runner
2
N= angular speed, h= net head
13. Specific speed-is a number used to predict the performance of the hydraulic turbines
a) In English units

Ns= 5 , where: N= angular speed in rpm, h= net head in feet
4
b) In SI Units
0.2623
Ns= 45
14. Hydraulic efficiency is the ratio of the utilized head to the net head

Eh =

Components Parts of a water turbine

A. Wheel- commonly known as runner, it is along with the vanes on its periphery, rotates under the action
of water gliding on the vanes comes from the gliding apparatus.
B. Guiding Apparatus (or mechanism), which guides the water to the vanes of runner.
C. In addition, there will be a source of supply from which water will come through a pipe line ( known as
penstock) to the guiding mechanism also, there will be a tailrace in which the water, after gliding over
the moving blades of the runner and passing out of runner, will ultimately fall from the turbine.

Classification of turbine according to the direction of flow

A. Radially inward flow turbine


B. Radially outward flow turbine
C. Axial flow turbine or parallel flow
D. Mixed flow turbine

A turbine, whether Impulse or Reaction may have one of the following settings
A. Vertical setting, B. Horizontal setting C. Above the tai race D. Below the tail race

A turbine is said to be vertical or with vertical setting when its shaft is vertical and its runner is horizontal,
this arrangement gives better efficiency due to the reduced losses. This setting is usually used for
reaction turbines, it may also used for pelton wheel having more than two nozzles.The vertical setting is
said to be used for low heads.
A turbine is said to be horizontal or with horizontal setting when its shaft is horizontal and its runner is
vertical. This setting is usually for Pelton wheel with one or two nozzles.
HEADS at Inlet and Outlet of Runner of Reaction Turbine

2
H= + 2, meters of water; Similarly, if V1 and P1 are respectively the velocity and intensity of pressure
at the outlet of runner, the total head at outlet runner,
2
H1 = 1 + 2 meters of water; where: h= total head at inlet, in m; V= absolute velocity with which water
from guide passages, enters the inlet of the runner, m/s; P= intensity of pressure at the inlet of the
runner in kg/m2. Applying modified Bernoullis Theorem to the outlet of the horizontal runner.
H= h1+ total losses between inlet and outlet of the runner
= h1 + friction loss(hydraulic loss) in runner + head used due to the work done by water on runner. If hf
be the loss of head in guide passages and Z be the elevation of inlet above the tail race, he total head H
on turbine
2
H= hf + + 2 + Z
Efficiency and horsepower of reaction turbine
The efficiency of a turbine may be
a) Hydraulic efficiency, h
b) Mechanical efficiency,m
c) Overall efficiency, o

Hydraulic efficiency = =
75

Where; 75
= horsepower of water on available water horsepower

75=1 metric horsepower,


Mechanical efficiency= =

Overall efficiency, 0 =m x h = =
Hydraaulic Turbines (Performance)
1) Specific Speed of turbine- speed of an imaginary or specific turbine( which is a small model of
the original turbine) which develops one SHP when working under a net head of one meter.
Discharge of turbine, Q= Area of flow times velocity of flow.= DbVf; taking the turbine to be a
reaction turbine , but;
(60)
U= tangential velocity = 60
; D=
; D, Speed ratio= of a turbine is a certain constant,
2
U= constant ( 2) : Constant =Cv= coefficient of velocity of nozzle

For a turbine, b =fD; therefore b

Ns= 5
4
Equation for specific speed using 2 jet pelton wheel
2
Ns2= 5
4
3. Conditions of working of a Turbine
1 1 2 2
=
1 2
1= 2
1 12 2 22
3 = 3 ,
12 22
1 12 2 22
5 = 5
12 22
1 1 2 2
; Under two conditions; Ns = 5 = 5
14 24

Example Problem #01


A reaction turbine develops 500 BHP. Flow through the turbine is 50 cfs. Water enters at 20 fps with
a 100 ft pressure head. The elevation of the turbine above the tail water level is 10 ft. Find the
effective head.
Solution:
2 2
h= +z + 2
(20)2 (0)2
= 100 + 10 +
2(32.2)
h= 116.2 ft
Example #02
A proposed scheme has an available head of 480 m and the 3 single jet pelton wheels to be installed
are required to run at 330 rpm. Assuming an over-all efficiency of 85%, determine the total quantity
of water required per second. Assume that the specific speed of Pelton is 18.
Solution:
H= 480 m N= 330 rpm, 0 =85%, Ns = 18

From Formula: Ns= 5 ;
4
()1.25 18(480)1.25
= = = 122.54
330
= 15018.28093 hp

=
15018.28093
= = = 0.85
= 17,668.56 HP

=
75
(75) 17668.56 (75)
= = 1000(480) = 2.76 m3/sec, For single jet Pelton wheel
= 2.76(3)= 8.28 m3/sec

Example #03
Determine the specific speed of a pelton wheel of which the basis design criteria are as follows:
Coefficient of velocity of nozzle, Cv= 0.98, runner velocity = 0.46 x jet velocity,
1
= =
10
Overall efficiency = 82%

Solution; for runner , tangential velocity, U= 60
eq. 1
Where: Dmean= effective runner diameter,m
N= speed of the runner, rpm
U= tangential velocity, m/s
For water jet, absolute velocity or jet velocity= V= Cv 2----eq.2
Where Cv =coefficient of velocity of nozzle
H= net head of turbine,m
V= jet velocity m/s, But runner velocity, U=0.46 x V----eq.3
Substituting eq.1 and eq.2 to eq.3

60
= 0.46 x Cv2; Cv= 0.98 (given)

60
= 0.46 x 0.98 2 9.81
38.135
=
75
= 0= =
= mass of striking water, kg/s
0
= 0 75 = 75
2
= = 4
x Cv 2
0.82 1000 2 0.982 9.81
SHP= 4 75
H
SHP= 37.275dc2x H3/4
Taking the square root of both sides
SHP1/2 = [ 37.275 dc2 x H3/2]1/2
= 6.1 dc H3/4
From Formula

Ns= 5 ; N= speed of the runner
4
38.135( )
6.1

Ns= 5
4
1 3
38.135 2 6.1 4 1
Ns= 5 ; but =
10
4
38.135 6.1 4
Ns= 5
10 4
Ns= 23.26 rpm

S-ar putea să vă placă și